GMAT Sample Essays

All GMAT students know the Argument essay comes first in the test, before we get to the more important Quantitative and Verbal sections, so it’s important not to exhaust ourselves in this first part of the exam. One way to stay fresh for the rest of the exam is to have a good idea of what a high-scoring essay looks like, so we have provided some GMAT sample essays to review. Below we look at a “6” and discuss why it would have likely received a perfect score, then we examine a “4” and discuss how it could have been strengthened. You can find more example essays in the GMAT Official Guide. Happy writing!

ESSAY QUESTION #1:

The following appeared in the editorial section of a national news magazine:

“The rating system for electronic games is similar to the movie rating system in that it provides consumers with a quick reference so that they can determine if the subject matter and contents are appropriate. This electronic game rating system is not working because it is self-regulated and the fines for violating the rating system are nominal. As a result an independent body should oversee the game industry and companies that knowingly violate the rating system should be prohibited from releasing a game for two years.”

Discuss how well reasoned you find this argument. Point out flaws in the argument’s logic and analyze the argument’s underlying assumptions. In addition, evaluate how supporting evidence is used and what evidence might counter the argument’s conclusion. You may also discuss what additional evidence could be used to strengthen the argument or what changes would make the argument more logically sound.

The author concludes that electronic game rating system is not working compared to the movie rating system. He gives reasoning for the argument by stating that electronic companies ability to self manage and regulate the rating system is part of the problem. Author also gives reason for what needs to be done in order for electronic rating system to work, but his reasons are weak. In the next few paragraphs, I will explain why the author’s reasons are weak and what could strengthen the argument.

First, the author mentions that by hiring an independent company to oversee the operation of the electronic rating system would solve the issue. This big assumption that author makes here is that he assumes independent company would do a better job in rating games than electronic game companies themselves. If the electronic gaming companies had a better understanding on how to improve, they might themselves do a better job than independent company to oversee the ratings.

Second, the author mentions that if an electronic company violates the rating system rule, then the penalty would be to prohibit that company from releasing any games for two years. This is another weak point the makes to support his argument because if the electronic company regular product-life cycle is to release each gave every two years, this penalty wouldn’t hurt the company at all. Also, there is not way of assessing how many year of prohibition would be adequate.

Author could have strengthen his argument, if he provided some data point such as from the movie industry to defend this stance that making independent company oversee and prohibition of movie release actually worked in the long-run. Perhaps, some research data that showed making these changes would actually work, would benefit the author’s overall argument stance.

Therefore, the author’s argument that electronic game rating system is not working is weak. Both of the points he made regarding independent company oversee and violation penalty are weak without data showing that it might work.

RECAP: The first thing that stands out in this essay is the organization. The paragraphs as clearly laid out and succinct, and each begin with a great transition word or phrase. The introductory paragraph, while unfortunately uses some unneeded self-reference, clearly demonstrates an understanding of the presented argument, which is mandatory of all “6” essays. Each flaw is then pointed out in a body paragraph, and the author then chooses to include a nice “how to strengthen” paragraph to demonstrate that he/she knows the argument at a more advanced level! The conclusion is clear, and reinforces the claims previously made. While no means perfect, the strong reasoning and clarity of organization definitely give this author a “6”!

ESSAY QUESTION #2:

Political organizations that advocate the use of violence to achieve their goals should be prohibited from operating within our country. Such groups are only interested in achieving short term goals which lead to more serious long term problems.

Political organizations that advocate the use of violence to achieve its goals can sometimes lead to destruction and devastation. However, that claim that such groups are detrimental to society does not follow the same line of reasoning. These groups might be of great help to certain sections of society. Also the claim that short term solutions can only lead to more serious long term problems is stated without any evidence. Hence the above argument is flawed.

Firstly, political organizations might be of great help to certain sections of society. Take for example the Indian National Congress party that helped tons of Indian’s voice their views to the British government during the British rule in india. Although this political organization advocated the use of violence, the organization was critical to India becoming a free of British rule in 1947.

Secondly, the argument assumes that short term solutions lead to more serious long term problems. Short term solutions are often very important in achieving long term goals, irrespective of whether the organizations advocate violence or not. The argument must also state clearly what constitutes violence. In a free country, such as the United States, to prohibit any political organization is to put a hold on their freedom. As long as the violence or aggressive behavior is not illegal, one can not prohibit a political organization from operating.

Thus although the argument seems to convey a valid point, that political organizations that advocate the use of violence should be prohibited from operating, it is worded to strongly and lacks evidence to support its claims that these groups are detrimental to society.

RECAP: The major difference between these two essays is length. More is not always better, but a student who can write 5-6 paragraphs within the given time frame definitely will stand out as a better planner, and as someone who already had a workable template. Furthermore, some minor grammar and spelling mistakes interfere with the essay’s clarity. The thesis, “Hence the above argument is flawed, “ is not as strongly worded as it could be, and doesn’t stand on its own as a powerful statement. The conclusion also seems to weaken the essay by admitting the argument conveys “a valid point.” This concession, without being more specific, weakens the overall thesis. The takeaway: length, specificity, and strength of argument will take this “4” to a “6.”

More AWA Resources: GMAT Essay Writing Guide

gmat essay answer examples

Homepage > GMAT IR and AWA > GMAT Analytical Writing Assessment tips – How to score a perfect 6 on GMAT AWA

GMAT Analytical Writing Assessment tips – How to score a perfect 6 on GMAT AWA

Posted by Suheb Hussain | Aug 6, 2020 | GMAT Focus Edition , GMAT IR and AWA , GMAT Preparation

GMAT Analytical Writing Assessment tips – How to score a perfect 6 on GMAT AWA

The GMAT Analytical Writing Assessment (AWA) section measures your ability to analyze an argument and communicate your thoughts or ideas. What you simply need to do is to critically analyze the reasoning given behind a given argument. To complete the AWA section of the GMAT you are allotted 30 minutes.

GMAT Analytical writing Assessment

Here is the outline of the article:

How to create a well rounded Analytical Writing Assessment essay?

  • How does ‘GMAT Write’ analyze your AWA essay?

GMAT AWA tips – GMAT Analytical Writing Assessment Template

  • Sample GMAT Analytical Writing Assessment essay

Let’s discuss now how you can score a perfect 6 on the GMAT Analytical Writing Assessment section. Here are a few tips to prepare you for the GMAT AWA section.

Step 1 –  Understand the process of creating a well-rounded analysis of an argument. You can do that in two ways or rather with the help of two tools.

  • AWA template by Chineseburned from gmatclub

Step 2 – Practice! Practice! Practice! the AWA questions. Here is a list of practice questions for you.

GMAT Write – An AWA practice tool by GMAC

GMAT Write is a writing tool provided by GMAC. It gives you access to 2 unique essay prompts and a chance to write 4 essays. The good thing about this tool is that it scores your essay based on the scoring algorithm used by the official GMAT exam. But, it costs $29.99 to subscribe to this tool.

Take a look at this article to know the importance of AWA and IR sections for business school admissions.

How does the ‘GMAT Write’ analyze your GMAT AWA essay?

Once you submit an essay, it scores you on 4 categories:

  • Analysis of the issue
  • Supports ideas
  • Organizes coherent idea
  • Language control

Based on these 4 categories it gives you an overall score for the essay.

Even if you don’t buy this tool, you still get to know what GMAT considers an ideal essay to be from it. An ideal essay should:

  • Identify and analyze significant flaws in the argument
  • Support the critique using relevant supporting reasons and/or examples
  • Be a clearly organized and coherent response
  • Demonstrate control of language, including diction, syntax, and conventions of standard written English

Source: https://www.mba.com/exam-prep/gmat-write

Once you know what factors make a perfect essay, the next thing you should do is creating a template. The most popular template for AWA is the one provided by ‘Chineseburned’ user on the gmatclub forum.

If you are planning to take the GMAT, we can help you with a personalized study plan and give you access to quality online content to prepare. Write to us at  [email protected] . We are the  most reviewed GMAT prep company on gmatclub with more than 1950 reviews. Why don’t you take a free trial and judge for yourself?

Here are the details of his template:

Structure of the essay

  • Introduction: Restate the argument and point out the flaws. Now, state your views which you’ll discuss in the next paragraphs.
  • First paragraph: State your first critique of the argument and support your view with an example.
  • Second paragraph: State your first critique of the argument and support your view with an example.
  • Third paragraph: Pose a few questions for the argument. The absence of information in the argument to answer your questions weaken it further.
  • Fourth argument: State information that you feel would have strengthened the argument but is absent. (This is an extra paragraph which is not in Chineseburned template)
  • Conclusion: State that the argument is flawed because of the above reasons and which reasons could have strengthened the argument.
Learn how the IR section is scored.

Sample GMAT Analytical Writing Assessment AWA essay

Here is a sample AWA essay question for practice. We have also provided the answer to this question based on the template.

The following appeared in an Excelsior Company memorandum. “The Excelsior Company plans to introduce its own brand of coffee. Since coffee is an expensive food item, and since there are already many established brands of coffee, the best way to gain customers for the Excelsior brand is to do what Superior, the leading coffee company, did when it introduced the newest brand in its line of coffees: conduct a temporary sales promotion that offers free samples, price reductions, and discount coupons for the new brand.” Discuss how well reasoned you find this argument. In your discussion, be sure to analyze the line of reasoning and the use of evidence in the argument. For example, you may need to consider what questionable assumptions underlie the thinking and what alternative explanations or counterexamples might weaken the conclusion. You can also discuss what sort of evidence would strengthen or refute the argument, what changes in the argument would make it more logically sound, and what, if anything, would help you better evaluate its conclusion.

GMAT AWA essay

The Excelsior company wants to introduce its own brand of coffee since coffee is an expensive food item and might have good profit margins. However, as there are several established brands of coffee already, it needs to work out a way to gain a foothold in the market. For this purpose, the company plans to pursue the same marketing strategy that the leading coffee company Superior had used. However, I think there are several flaws in that line of reasoning. And hence this argument seems a little weak.

First, Superior is already an established brand and the market leader. No data is given as to how long back it entered the coffee market. Without this information, it is difficult to ascertain the utility of the marketing strategy for Excelsior. A brand might take decades to establish, and what worked a decade back, need not work today. For example, when Google was initially set up, it depended on word of mouth, rather than any communication media, for its marketing. As it was initially just an internal product for the University, this approach worked for them. However, it might not work for any company now. We see these days that most new web companies do advertise heavily when they initially launch a product for the masses.

Second, no information has been provided about the target market for the coffee or the pricing strategy. It might be that the coffee produced by Superior is one of the cheaper products in the market. And that they earn their profits from volume sales rather than having a niche market. Whereas, Excelsior might be aiming at the niche market of expensive and luxury coffee. If that were the case, then the same marketing strategies might not be relevant for Excelsior. Can Hyundai apply the same approach towards the market as does a luxury brand like Ferrari?

Without some of the relevant and contextual details, it is difficult to judge the efficacy of using the same marketing strategy that a competitor applied some unknown years back. If we were to get some more details in this regard, we could have judged the situation better.

Although all the factors being the same, it might not be a bad idea to follow the same approach as that of a leading brand in the market. Sometimes there is nothing wrong with sticking to the tried and the tested. In the absence of any other better idea, Excelsior might not have much to lose if it pursues the same methods as did Superior. It’s better to have some plan rather than no plan.

Conducting a temporary sales promotion that offers free samples, price reductions and discount coupons might not be such a bad idea. It could be a good ploy to get users to try out the product. And discounts and other monetary incentives are usually very effective in this regard. If the quality of the product is good enough and the pricing is appropriate, introducing enough customers to the product through such offers might give a significant push to consumer interest in the product. While customer satisfaction might leverage word of mouth marketing.

Therefore, depending on the context the approach might or might not be a beneficial one. This essay question has been taken from gmatclub and the answer has been judged as a perfect 6 by the GMAT Write tool. The answer was written by the user ‘rish2jain’ from gmatclub forum.

To score a perfect 6 on the GMAT Analytical Writing Assessment section, it is important that you learn the AWA template and practice as much as you can. Just don’t overdo it. It is a good practice to devote 10% of your preparation time to GMAT Analytical Writing Assessment section.

About The Author

gmat essay answer examples

Suheb Hussain

Don't limit yourself, don't miss these.

gmat essay answer examples

Helpful posts Curated just for you!

gmat essay answer examples

You might also like

gmat essay answer examples

Browse Related Topics

gmat essay answer examples

Achieve 685+ on the GMAT in 30 days!

Sign up for our free trial and get.

gmat essay answer examples

400+ Practice questions with detailed solutions

10+ hours of ai-driven video lessons, adaptive mock test with osr+ analysis.

Begin your GMAT Prep today!

gmat essay answer examples

How to write your GMAT AWA Essay [Effectively]

gmat essay answer examples

INTRODUCTION

Hello there!

If you’ve found your way to this blog, we assume that you have begun to think about that often-neglected section of the GMAT – the AWA!

If wishes could come true, we’re guessing that GMAT aspirants would have wished away the AWA section away by now!

The AWA involves staring at a blank page and cursor and thinking up stuff to write, not an easy task for many people.

It also consumes a lot of mental bandwidth at the very beginning of the test.

To top it off, it does not even add to your final score!

We can understand why it isn’t your favorite section on the GMAT 🙂

But here’s the good news! There are not many shortcuts to mastering GMAT Quant and Verbal, but there are shortcuts to cracking the AWA section.

By the time you finish reading this guide, you will know what these hacks are. You will learn how to write an effective AWA essay that gets you a good score and leaves you charged for the real test that lies ahead.

This blog will teach you –

  • What you need to know BEFORE you start preparing for the AWA
  • How to use the 30 minutes allotted to AWA to maximum effect
  • How to use a template to make the AWA writing process simpler

Besides this, you will find 8 sample AWA essays to observe and learn from.

Happy reading! 🙂

7 Things to remember before you start prepping for the AWA

gmat essay answer examples

Before you dive into AWA preparation, there are a few things you should know about the AWA. Many of these facts will ease your AWA fears and bring a smile to your face!

1. Why is the AWA section on the GMAT?

Each section of the GMAT is carefully constructed ( at the expense of millions of dollars, we kid you not!), to test your readiness for an MBA program and for your post-MBA career. One of the skills you will definitely need post-MBA is an ability to analyze an argument impartially and convey your perspective clearly.

This is what the AWA tests you on.

2. On the AWA, you need to be a lawyer, that is, you need to find faults with the given argument. You do not need to be a journalist, that is, you do not need to write about all possible perspectives of an issue.

Also remember, that this is an analysis, not an opinion piece. Do not bring YOUR perspective and your opinions into the essay. Your only goal is to analyse the given argument.

3. Unlike the Quant and Verbal sections, where your thumb-rule should be to get as high a score as possible, we’d suggest that you do not expend too much mental energy on the AWA, trying to score a 6 on 6.

Getting a perfect 6 will look good on your GMAT score card, and will sound great as you’re telling your friends about it. But it will not be the make-or-break factor in your application. A 4 or a 5 is good enough.

4. The AWA is graded by an E-reader application and by a human reader. Since there is an element of automated grading, you can rig the test to an extent. We’ve found that if you write a substantial essay of over 500 words, and if you structure the argument well ( check our CrackVerbal template in the following chapter), you are almost guaranteed to get a 4+ score!

5. Assuming that you prepare for the GMAT over a course of three months, we recommend that you practice writing 5 to 10 essays, and make sure you get feedback for all of them. If you cover this much practice ground, you’re good to go!

6. One of the best things about the AWA section is that you know all of the questions beforehand (yes, they’re all up there on the GMAC site – Analytical Writing Section ). So you do not have to go hunting for ‘authentic’ AWA essay questions.

7. Now you can choose the order in which you want to take up the sections before starting the test. It is advisable to keep in mind the order that would be helpful for you and prepare for the AWA based on that strategy. This is a recent change to the GMAT test structure. It was introduced in July 2017. We have done a detailed analysis of what this means to an Indian GMAT test-taker in the this blog

8. We saved the coolest point for last 🙂

The AWA lends itself very easily to the use of an essay template. No matter what the argument prompt is, you can bet that there will be at least 3 glaring errors of logic in it. You can, therefore, use a template to structure your AWA essay. Using a template takes most of the stress away from the AWA section.

In the few minutes before you start, you can jot down the template on your scratchpad, so that you don’t have to remember it anymore. Also, because you can plan many of your sentences beforehand, you can get at least a 100 words down before you even read the question!

There are a lot of templates on the internet – probably the most famous one being the Chineseburned AWA template.

At CrackVerbal, we have our own template for the AWA, a modified version of the Chineseburned template. We call it the CrackVerbal AWA Template on Steroids! 🙂

The AWA Writing Process

MBA Essay Writing

1. Write your templatized response

This should take you about 5 minutes:

Type out your prepared template response. Below is a sample. We definitely do not recommend that you use the same words. What you can do, however, is read a few templates on the net, and then write your own. Since you have written it yourself, it will be that much easier to memorise it.

CRACKVERBAL AWA TEMPLATE

The argument claims that < restate the argument >. Stated in this way the argument fails to take into account a few key factors which could call the conclusion to question. It rests on some assumptions, for which there is no clear evidence. Therefore, the argument is unconvincing and falls apart at the seams.

Paragraph 1:

1. Firstly, ( ) 2. This statement is a stretch and not substantiated in any way. 3. The argument would have been much clearer if ( )

Paragraph 2:

1. Second ( ). This is again a very weak and unsupported claim as the argument ( ). 2. For example, 3. This argument would have sounded a lot more convincing if 4. In addition, it would have been strengthened ever further if the argument provided evidence that

Paragraph 3:

1. Finally, the argument concludes that 2. However, what is not clear here is ( ) 3. If there had been evidence to support ( )

In summary, the argument fails to convince because of the faulty assumptions aforementioned. If the argument had drawn upon examples as suggested, and thereby plugged in the holes in the reasoning, it would have been far sounder on the whole.

2. Brainstorm

Now that you have put the pre-planned portion of the essay down, it’s time to read the AWA prompt and wear your thinking hat. GMAT, in its politically correct, non-partisan way, says ‘Discuss how well-reasoned you find this argument’. Remember however, that an AWA argument is never well-reasoned!

There are always a couple of glaring flaws in logic you can pounce on. If these flaws do not occur to you immediately, because of test-day stress, do not assume that you have been given a particularly sound argument. There is no such thing on the AWA!

If you’re unable to be critical, imagine that the author of the argument is somebody you dislike..a teacher you hated at college, or that guy who overtook you and almost dented your car this morning! There, now you’re in the right frame of mind to attack the argument 🙂

Before you do so, you need to understand the three elements of the argument – Conclusion, Premise and Assumptions.

Let us look at an example, and detect these three elements.

“Most companies would agree that as the risk of physical injury occurring on the job increases, the wages paid to employees should also increase. Hence it makes financial sense for employers to make the workplace safer: they could thus reduce their payroll expenses and save money.”

The conclusion is the decision/statement that the author has arrived at. In this case, the conclusion is the last sentence – “Hence it makes financial sense for employers to make the workplace safer: they could thus reduce their payroll expenses and save money.”

The premises are the building blocks of facts on which the conclusion rests. In other words, a premise is what is offered as support for the conclusion. In this case, the premise is – Most companies would agree that as the risk of physical injury occurring on the job increases, the wages paid to employees should also increase.

Assumptions are the unstated, unwritten premises that plug the gap between the written premises and the conclusion. It is the assumptions that you need to attack on the AWA!

How do you attack assumptions?

Remember that an assumption can be incorrect for a variety of reasons. Here are a few types of incorrect assumptions –

1. The Sampling Assumption – The sampling argument assumes that a small group is representative of a much larger group to which it belongs.

2. The illogical analogy assumption – The illogical analogy states that because something applies to A, it applies to B also.

3. The Causal Assumption – The Causal Assumption confuses correlation with causation. That means, just because ‘A’ usually occurs after ‘B’ occurs, does not necessarily imply that B happens because of A.

4. The Data Bias – This occurs when the data for a statistical inference itself is drawn from a sample that is not representative of the population under consideration. This is a case of faulty data leading to faulty assumptions.

5. The Non Sequitur – This simply means, finding a connection where there is none. Non Sequitur means “does not follow,” which is short for: the conclusion does not follow from the premise.

Don’t let these categories overwhelm you. We’ve put them down here to get you thinking. However, you can find faulty assumptions with ease, even if you have no clue what a non sequitur is!

As you brainstorm, you will need to jot down your thoughts on the scratchpad. Keep it crisp and brief. Make sure you have these things down –

1. Conclusion + Premise: 2. Flawed Assumption #1: 3. Flawed Assumption #2: 4. Flawed Assumption #3:

For each assumption, also make a cursory note of why it is flawed, an example that talks about why it is flawed, and what additional data would strengthen the argument ( or if you are convinced that you can remember these additional details without having to make a note of them, you can get on with the writing! )

This should take you about 15 minutes:

Here is where you fill in your templatized response with specific details.

The only detail you need to add to the first paragraph is a summary of the argument that is presented. In the above template, your summary should go here ->

1. Start off by pointing out the first flawed assumption. 2. Explain why this assumption is flawed. 3. Give an example that supports the flaw. 4. Explain what further information could have strengthened this argument.

1. Start off by pointing out the second flawed assumption. 2. Explain why this assumption is flawed. 3. Give an example that supports the flaw. 4. Explain what further information could have strengthened this argument.

Paragraph 4:

1. Start off by pointing out the third flawed assumption. 2. Explain why this assumption is flawed. 3. Give an example that supports the flaw. 4. Explain what further information could have strengthened this argument.

Paragraph 5:

This is the concluding paragraph. You already have it down in your template! 🙂

4. Proofread

Are you wondering if three minutes is really enough time to proof-read a 500 word essay?

Here’s the deal – The AWA section is about whether you can analyse an argument and discuss it in an articulate manner. It is not a test of grammar and spelling. Hence, the GMAT will excuse minor errors in spelling and grammar.

However, you should understand that a human reader is going to be reviewing your work, and any human reader will have an unconscious bias against bad grammar and spellings. Hence, you want to keep your essay as error-free as possible, without worrying about it too much.

Three minutes should be able time for you to quickly glance through the document and make sure you haven’t made any obvious errors.

Voila! 🙂 Your AWA essay is ready!

Also Read: GMAT Section Selection – Everything you need to know

Sample AWA Essays

gmat essay answer examples

Sample Essay 1

”Most companies would agree that as the risk of physical injury occurring on the job increases, the wages paid to employees should also increase. Hence it makes financial sense for employers to make the workplace safer: they could thus reduce their payroll expenses and save money.”

Discuss how well reasoned you find this argument. In your discussion be sure to analyze the line of reasoning and the use of evidence in the argument.

For example, you may need to consider what questionable assumptions underlying the thinking and what alternative explanations or counter examples might weaken the conclusion. You can also discuss what sort of evidence would strengthen or refute the argument, what changes in the argument would make it more logically sound, and what, if anything, would help you better evaluate its conclusion.

Introduction:

This argument states that it makes financial sense for employers to make the workplace safer because lower wages could then be paid to employees. This conclusion is based on the premise that as the risk of physical injury increases, the wages paid to employees should also increase. However, this argument makes several unsupported assumptions. For example, the argument assumes that the costs associated with making the workplace safe do not outweigh the increased payroll expenses due to hazardous conditions.

Body Paragraph 1

The first issue to be addressed is whether increased labor costs justify large capital expenditures to improve the work environment. Clearly one could argue that if making the workplace safe would cost an exorbitant amount of money in comparison to leaving the workplace as is and paying slightly increased wages than it would not make sense to improve the work environment. For example, if making the workplace safe would cost $100 million versus additional payroll expenses of only $5,000 per year, it would make financial sense to simply pay the increased wages. No business or business owner would pay all that extra money just to save a couple dollars and improve employee health and relations. To consider this, a cost benefit analysis must be made. I also feel that although a cost benefit analysis should be the determining factor with regard to these decisions making financial sense, it may not be the determining factor with regard to making social, moral and ethical sense.

Body Paragraph 2

Finally one must understand that not all work environments can be made safer. For example, in the case of coal mining, a company only has limited ways of making the work environment safe. While companies may be able to ensure some safety precautions, they may not be able to provide all the safety measures necessary. In other words, a mining company has limited ability to control the air quality within a coal mine and therefore it cannot control the risk of employees getting black. In other words, regardless of the intent of the company, some jobs are simply dangerous in nature.

In conclusion, while at first it may seem to make financial sense to improve the safety of the work environment sometimes it truly does not make financial sense. Furthermore, financial sense may not be the only issue a company faces. Other types of analyses must be made such as the social ramifications of an unsafe work environment and the overall ability of a company to improve that environment (i.e., coal mine). Before any decision is made, all this things must be considered, not simply the reduction of payroll expenses.

Sample Essay 2

The following appeared in a memorandum issued by a large city’s council on the arts.

“In a recent citywide poll, fifteen percent more residents said that they watch television programs about the visual arts than was the case in a poll conducted five years ago. During these past five years, the number of people visiting our city’s art museums has increased by a similar percentage. Since the corporate funding that supports public television, where most of the visual arts programs appear, is now being threatened with severe cuts, we can expect that attendance at our city’s art museums will also start to decrease. Thus some of the city’s funds for supporting the arts should be reallocated to public television.”

Discuss how well reasoned… etc.

Introduction

In this argument the author concludes that the city should allocate some of its arts funding to public television. The conclusion is based on two facts: (1) attendance at the city’s art museum has increased proportionally with the increases in visual-arts program viewing on public television, and (2) public television is being threatened by severe cuts in corporate funding. While this argument is somewhat convincing, a few concerns need to be addressed.

To begin with, the argument depends on the assumption that increased exposure to the visual arts on television, mainly public television, has caused a similar increase in local art-museum attendance. However, just because increased art-museum attendance can be statistically correlated with similar increases in television viewing of visual-arts programs, this does not necessarily mean that the increased television viewing of arts is the cause of the rise in museum attendance.

Moreover, perhaps there are other factors relevant to increased interest in the local art museum; for instance, maybe a new director had procured more interesting, exciting acquisitions and exhibits during the period when museum attendance increased, in addition, the author could be overlooking a common cause of both increases. It is possible that some larger social or cultural phenomenon is responsible for greater public interest in both television arts programming and municipal art museums.

Body Paragraph 3

To be fair, however, we must recognize that the author’s assumption is a special case of a more general one that television viewing affects people’s attitudes and behavior. Common sense and observation tell me that this is indeed the case. After all, advertisers spend billions of dollars on television ad time because they trust this assumption as well.

In conclusion, I am somewhat persuaded by this author’s line of reasoning. The argument would be strengthened if the author were to consider and rule out other significant factors that might have caused the increase in visits to the local art museum.

Sample Essay 3

The following appeared in a report presented for discussion at a meeting of the directors of a company that manufactures parts for heavy machinery.

“The falling revenues that the company is experiencing coincide with delays in manufacturing. These delays, in turn, are due in large part to poor planning in purchasing metals. Consider further that the manager of the department that handles purchasing of raw materials has an excellent background in general business, psychology, and sociology, but knows little about the properties of metals. The company should, therefore, move the purchasing manager to the sales department and bring in a scientist from the research division to be manager of the purchasing department.”

In response to a coincidence between falling revenues and delays in manufacturing, the report recommends replacing the manager of the purchasing department. The grounds for this action are twofold. First, the delays are traced to poor planning in purchasing metals. Second, the purchasing manager’s lack of knowledge of the properties of metals is thought to be the cause of the poor planning. It is further recommended that the position of the purchasing manager be filled by a scientist from the research division and that the current purchasing manager be reassigned to the sales department. In support of this latter recommendation, the report states that the current purchasing manager’s background in general business, psychology, and sociology equip him for this new assignment. The recommendations advanced in the report are questionable for two reasons.

To begin with, the report fails to establish a causal connection between the falling revenues of the company and the delays in manufacturing. The mere fact that falling revenues coincide with delays in manufacturing is insufficient to conclude that the delays caused the decline in revenue. Without compelling evidence to support the causal connection between these two events, the report’s recommendations are not worthy of consideration.

Second, a central assumption of the report is that knowledge of the properties of metals is necessary for planning in purchasing metals. No evidence is stated in the report to support this crucial assumption. Moreover, it is not obvious that such knowledge would be required to perform this task. Since planning is essentially a logistical function, it is doubtful that in-depth knowledge of the properties of metals would be helpful in accomplishing this task.

In conclusion, this is a weak argument. To strengthen the recommendation that the manager of the purchasing department be replaced, the author would have to demonstrate that the falling revenues were a result of the delays in manufacturing. Additionally, the author would have to show that knowledge of the properties of metals is a prerequisite for planning in purchasing metals.

Sample Essay 4

The following appeared in an announcement issued by the publisher of The Mercury, a weekly newspaper.

“Since a competing lower-priced newspaper, The Bugle, was started five years ago, The Mercury’s circulation has declined by 10,000 readers. The best way to get more people to read The Mercury is to reduce its price below that of The Bugle, at least until circulation increases to former levels. The increased circulation of The Mercury will attract more businesses to buy advertising space in the paper.”

A newspaper publisher is recommending that the price of its paper, The Mercury, be reduced below the price of a competing newspaper, The Bugle. This recommendation responds to a severe decline in circulation of The Mercury during the 5-year period following the introduction of The Bugle. The publisher’s line of reasoning is that lowering the price of The Mercury will increase its readership, thereby increasing profits because a wider readership attracts more advertisers. This line of reasoning is problematic in two critical respects.

While it is clear that increased circulation would make the paper more attractive to potential advertisers, it is not obvious that lowering the subscription price is the most effective way to gain new readers. The publisher assumes that price is the only factor that caused the decline in readership. But no evidence is given to support this claim. Moreover, given that The Mercury was the established local paper, it is unlikely that such a mass exodus of its readers would be explained by subscription price alone.

There are many other factors that might account for a decline in The Mercury’s popularity. For instance, readers might be displeased with the extent and accuracy of its news reporting, or the balance of local to other news coverage. Moreover, it is possible The Mercury has recently changed editors, giving the paper a locally unpopular political perspective. Or perhaps readers are unhappy with the paper’s format, the timeliness of its feature articles, its comics or advice columns, the extent and accuracy of its local event calendar, or its rate of errors.

In conclusion, this argument is weak because it depends on an oversimplified assumption about the causal connection between the price of the paper and its popularity. To strengthen the argument, the author must identify and explore relevant factors beyond cost before concluding that lowering subscription prices will increase circulation and, thereby, increase advertising revenues.

Sample Essay 5

The following appeared as part of an article in a magazine devoted to regional life.

“Corporations should look to the city of Helios when seeking new business opportunities or a new location. Even in the recent recession, Helios’s unemployment rate was lower than the regional average. It is the industrial center of the region, and historically it has provided more than its share of the region’s manufacturing jobs. In addition, Helios is attempting to expand its economic base by attracting companies that focus on research and development of innovative technologies.”

In this argument corporations are urged to consider the city of Helios when seeking a new location or new business opportunities. To support this recommendation, the author points out that Helios is the industrial center of the region, providing most of the region’s manufacturing jobs and enjoying a lower-than-average unemployment rate. Moreover, it is argued, efforts are currently underway to expand the economic base of the city by attracting companies that focus on research and development of innovative technologies. This argument is problematic for two reasons.

To begin with, it is questionable whether the available labor pool in Helios could support all types of corporations. Given that Helios has attracted mainly industrial and manufacturing companies in the past, it is unlikely that the local pool of prospective employees would be suitable for corporations of other types. For example, the needs of research and development companies would not be met by a labor force trained in manufacturing skills. For this reason, it’s unlikely that Helios will be successful in its attempt to attract companies that focus or research and development of innovative technologies.

Another problem with the available work force is its size. Due to the lower than average unemployment rate in Helios, corporations that require large numbers of workers would not find Helios attractive. The fact that few persons are out of work suggests that new corporations will have to either attract new workers to Helios or pay the existing workers higher wages in order to lure them away from their current jobs. Neither of these alternatives seems enticing to companies seeking to relocate.

In conclusion, the author has not succeeded in providing compelling reasons for selecting Helios as the site for a company wishing to relocate. In fact, the reasons offered function better as reasons for not relocating to Helios. Nor has the author provided compelling reasons for companies seeking new business opportunities to choose Helios.

Sample Essay 6

The following appeared in the health section of a magazine on trends and lifestyles.

“People who use the artificial sweetener aspartame are better off consuming sugar, since aspartame can actually contribute to weight gain rather than weight loss. For example, high levels of aspartame have been shown to trigger a craving for food bydepleting the brain of a chemical that registers satiety, or the sense of being full. Furthermore, studies suggest that sugars, if consumed after at least 45 minutes of continuous exercise, actually enhance the body’s ability to burn fat. Consequently, those who drink aspartame-sweetened juices after exercise will also lose this calorie-burning benefit. Thus it appears that people consuming aspartame rather than sugar are unlikely to achieve their dietary goals.”

In this argument the author concludes that people trying to lose weight are better off consuming sugar than the artificial sweetener aspartame. To support this conclusion the author argues that aspartame can cause weight gain by triggering food cravings, whereas sugar actually enhances the body’s ability to burn fat. Neither of these reasons provides sufficient support for the conclusion.

The first reason that aspartame encourages food cravings is supported by research findings that high levels of aspartame deplete the brain chemical responsible for registering a sense of being satedHidden text (sated, sating ), or full. But the author’s generalization based on this research is unreliable. The research was based on a sample in which large amounts of aspartame were administered; however, the author applies the research findings to a target population that includes all aspartame users, many of whom would probably not consume high levels of the artificial sweetener.

The second reason that sugar enhances the body’s ability to burn fat is based on the studies in which experimental groups, whose members consumed sugar after at least 45 minutes of continuous exercise, showed increased rates of fat burning. The author’s general claim, however, applies to all dieters who use sugar instead of aspartame, not just to those who use sugar after long periods of exercise. Once again, the author’s generalization is unreliable because it is based on a sample that clearly does not represent all dieters.

To conclude, each of the studies cited by the author bases its findings on evidence that does not represent dieters in general; for this reason, neither premise of this argument is a reliable generalization. Consequently, I am not convinced that dieters are better off consuming sugar instead of aspartame.

Sample Essay 7

The following appeared in the editorial section of a corporate newsletter.

“The common notion that workers are generally apathetic about management issues is false, or at least outdated: a recently published survey indicates that 79 percent of the nearly 1,200 workers who responded to survey questionnaires expressed a high level of interest in the topics of corporate restructuring and redesign of benefits programs.”

Based upon a survey among workers that indicates a high level of interest in the topics of corporate restructuring and redesign of benefits programs, the author concludes that workers are not apathetic about management issues. Specifically, it is argued that since 79 percent of the 1200 workers who responded to survey expressed interest in these topics, the notion that workers are apathetic about management issues is incorrect. The reasoning in this argument is problematic in several respects.

First, the statistics cited in the editorial may be misleading because the total number of workers employed by the corporation is not specified. For example, if the corporation employs 2000 workers, the fact that 79 percent of the nearly 1200 respondents showed interest in these topics provides strong support for the conclusion. On the other hand, if the corporation employs 200,000 workers, the conclusion is much weaker.

Another problem with the argument is that the respondents’ views are not necessarily representative of the views of the work force in general. For example, because the survey has to do with apathy, it makes sense that only less apathetic workers would respond to it, thereby distorting the overall picture of apathy among the work force. Without knowing how the survey was conducted, it is impossible to assess whether or not this is the case.

A third problem with the argument is that it makes a hasty generalization about the types of issues workers are interested in. It accords with common sense that workers would be interested in corporate restructuring and redesign of benefits programs, since these issues affect workers very directly. However, it is unfair to assume that workers would be similarly interested in other management issues—ones that do not affect them or affect them less directly.

In conclusion, this argument is not convincing as it stands. To strengthen it, the author would have to show that the respondents account for a significant and representative portion of all workers. Additionally, the author must provide evidence of workers’ interest other management topics—not just those that affect workers directly.

Sample Essay 8

The following appeared in the opinion column of a financial magazine.

“On average, middle-aged consumers devote 39 percent of their retail expenditure to department store products and services, while for younger consumers the average is only 25 percent. Since the number of middle-aged people will increase dramatically within the next decade, department stores can expect retail sales to increase significantly during that period. Furthermore, to take advantage of the trend, these stores should begin to replace some of those products intended to attract the younger consumer with products intended to attract the middle-aged consumer.”

Based on an expected increase in the number of middle-aged people during the next decade, the author predicts that retail sales at department stores will increase significantly over the next ten years. To bolster this prediction, the author cites statistics showing that middle-aged people devote a much higher percentage of their retail expenditure to department-store services and products than younger consumers do. Since the number of middle-aged consumers is on the rise and since they spend more than younger people on department-store goods and services, the author further recommends that department stores begin to adjust their inventories to capitalize on this trend. Specifically, it is recommended that department stores increase their inventory of products aimed at middle- aged consumers and decrease their inventory of products aimed at younger consumers. This argument is problematic for two reasons.

First, an increase in the number of middle-aged people does not necessarily portend an overall increase in department-store sales. It does so only on the assumption that other population groups will remain relatively constant. For example, if the expected increase in the number of middle-aged people is offset by an equally significant decrease in the number of younger people, there will be little or no net gain in sales.

Second, in recommending that department stores replace products intended to attract younger consumers with products more suitable to middle-aged consumers, the author assumes that the number of younger consumers will not also increase. Since a sizable increase in the population of younger consumers could conceivably offset the difference in the retail expenditure patterns of younger and middle- aged consumers, it would be unwise to make the recommended inventory adjustment lacking evidence to support this assumption.

This argument is unacceptable. To strengthen the argument the author would have to provide evidence that the population of younger consumers will remain relatively constant over the next decade.

We hope that our strategies help you conquer GMAT AWA with enough and more energy to spare for the sections that follow!

Now that you’ve figured out how to tackle the AWA section, do you want to put theory to practice and get your AWA essay graded?

Our experts here at CrackVerbal will evaluate and grade your AWA essay and give you specific, actionable feedback.

gmat essay answer examples

Talk To Your Advisor

RO București

Recently viewed courses

Recently viewed.

Find Your Dream School

This site uses various technologies, as described in our Privacy Policy, for personalization, measuring website use/performance, and targeted advertising, which may include storing and sharing information about your site visit with third parties. By continuing to use this website you consent to our Privacy Policy and Terms of Use .

   COVID-19 Update: To help students through this crisis, The Princeton Review will continue our "Enroll with Confidence" refund policies. For full details, please click here.

GMAT Sample Questions

Hidden laptop with hands typing, displaying a test diagram above the keyboard

Want a preview of the question types you'll face on the GMAT? Try your hand at the GMAT practice questions below. Then, check your answers against our in-depth explanations to see how you did.

We pulled these GMAT sample questions from our book Cracking the GMAT and from our test prep course materials. For more GMAT practice, take a full-length practice test with us held under the same testing conditions as the real thing. Find out how you'd score, and get  a personalized score report from us that shows your strengths and weaknesses.

  • GMAT Verbal Questions 
  • GMAT Math Questions
  • GMAT Integrated Reasoning Questions 
  • Essay Prompt 

Below you'll find sample GMAT questions covering the three question types you'll encounter on the Verbal section: Sentence Correction , Critical Reasoning, and Reading Comprehension.

GMAT Sentence Correction Questions

1. In order to better differentiate its product from generic brands, the cereal company first hired a marketing firm that specializes in creating campaigns to build brand awareness and then retools its factory to produce a variety of different shapes of cereal. (A) then retools its factory to produce a variety of different shapes of cereal (B) retools its factory to produce a variety of different shapes of cereal (C) then retooled its factory to produce a variety of different shapes of cereal (D) then will retool its factory to produce a variety of different shapes of cereal (E) then produces a variety of different shapes of cereal through retooling its factory

Answer: (C) The actions of the cereal company are not in parallel form. First the company hired then it retools . Eliminate choice (A). Choice (B) still has the same error. Choice (D) changes the verb form incorrectly to the future tense. Choice (E) rewrites the sentence but retains the error.

[+] See the Answer

2. Believed to be one of the first widely read female authors of the Western world, Christine de Pizan's masterwork The Book of the City of the Ladies , was written in 1405 and is a history of the Western world from the woman's point-of-view. (A) Believed to be one of the first widely read female authors of the Western world (B) Written by one of the first widely read female authors of the Western world (C) One of the first widely read female authors of the Western world, as some believe (D) Written by what some believe as one of the first widely read female authors of the Western world (E) Believed by some as one of the first works by a widely read female author in the Western world

Answer: (B) As written, this sentence has a misplaced modifier error: the book, The Book of the City of the Ladies isn't believed by anyone to be an author— Christine de Pizan is. Choices (A) and (C) repeat that error and can be eliminated. Choices (B) and (D) both change the introductory phrase to clearly refer to a written work, but choice (D) uses the incorrect idiom believe as instead of the correct form, believe to be . Choice (E) repeats that idiom error.

GMAT Critical Reasoning Questions

1. One food writer wrote that reducing the amount of animal products in one's diet can contribute to better health and well-being. Based on this claim, some people are completely eliminating meat from their diets in order to be healthier. The argument above relies on which of the following assumptions?

Answer: (B) The argument states that some people are eliminating meat from their diets because reducing the amount of animal products in one's diet can lead to better health. Meat is only one type of animal product, however. The argument assumes that by eliminating meat, the people are reducing the total amount of animal products in their diets. Choice (A) addresses increasing the amount of vegetables and grains, but the argument just deals with animal products. Choice (B) correctly addresses the people who are eliminating meat and states that those people are not increasing their consumption of dairy, which is another instance of using animal products. Thus, these people are actually reducing the amount of animal products in their diets. Choice (C) addresses most food writers, who are irrelevant to this argument. Choice (D) addresses health lifestyles, which are irrelevant to this particular argument. Choice (E) addresses the reasons behind not eating animal products, which is irrelevant to the argument.

2. Studies reveal that a daily exercise regimen helps stroke survivors regain dexterity in their extremities. Being given an exercise routine and having a consultation with a doctor about the exercise routine have been shown to be effective mechanisms to get patients to exercise daily. From the above information, which of the following statements can be reasonably inferred? (A) A stroke survivor that is given a detailed exercise plan and consults her physician about the plan will regain full dexterity in her extremities. (B) If a stroke survivor is not given an exercise plan and does not consult with a doctor, she will not regain dexterity in her extremities. (C) Stroke survivors who are given an exercise routine and consult with a doctor about that routine will sometimes regain dexterity in their extremities. (D) Being given an exercise routine and having a consultation with a doctor about the routine is the best way to help a stroke survivor regain dexterity in their extremities. (E) Only being given an exercise routine is necessary to regenerate dexterity in the extremities of seniors who have suffered a stroke.

Answer: (C) This is an inference question, so evaluate the passage and then look for an answer choice that can be reasonably inferred from the information. The passage states that a daily exercise regimen helps stroke survivors regain dexterity in their extremities and that survivors who are given an exercise routine and who have a consultation with a doctor about the routine have been shown to be effective at getting patients to exercise daily . So it can be inferred that if a survivor is given a routine and consults with a doctor, they are more likely to exercise daily, which will help them regain dexterity. Choice (A) is an example of extreme language. The phrasing will regain full dexterity is not promised in the information in the passage, as the passage only states that a routine and consultations may help a survivor exercise more. Eliminate (A). Choice (B) is also an example of extreme language. There is no way to discern from the information provided that a strong survivor would not regain dexterity without an exercise routine and a consultation, so eliminate (B). Choice (C) is a reasonable inference to make from the information in the passage so keep (C). Choice (D) also contains the extreme language best way . The information does not compare this method with any other method so eliminate (D). Choice (E) is recycled language and does not address consulting with a doctor so eliminate (E). The correct answer is (C).

GMAT Reading Comprehension Questions

Although oft-maligned in modern culture, the pigeon once stood not only for speed and reliability but also for grace and beauty. Darwin himself became a pigeon fancier after beginning to work with the humble Columbia livia , discovering them to be more fascinating than he had formerly believed. During the Victorian age, in fact, raising show pigeons was a popular hobby, with new breeds continuously arising as amateur (and not-so-amateur) ornithologists crossed animals in the hopes of creating ever more fantastic creatures. One of the most sought-after varieties was known as the Almond Tumbler, a name presumably derived from the color of the birds combined with the distinctive flight style. Over the course of many generations, this bird was so manipulated as to have a beak so small as to prevent the adult birds from feeding their offspring. And yet, it was wildly popular, drawing high prices at auctions and high prizes at competitions. How then did an animal once so well-loved come to be so loathed? As recently as World War II, the military used pigeons to carry messages but today, many people would kick a pigeon before they would feed one. Perhaps it is just a problem of population density - a lack of esteem for that which is ubiquitous. Pigeons have become our constant urban companions and, as such, have been transformed from symbols of peace, plenty, and prosperity, to representatives of disease and decay.

1. The primary purpose of this passage is to (A) convince the reader of the nobility of the pigeon, based on its history as a symbol of virtue (B) dissuade the reader from mistreating a once-majestic animal that has fallen from favor (C) rebut claims that the pigeon carries disease any more frequently than do other domestic animals (D) promote a renewal of pigeon fancying and a resurgence of breeds such as the Almond Tumbler (E) suggest that there might be more to the story of some urban wildlife than is commonly known

Answer: (E) The passage gives a brief description of the pigeon's place in recent human history and then goes on to contrast that with modern perspectives of the birds. Choice (A) goes too far—the author doesn't give any indication of believing the pigeon to be noble. Choice (B) focuses too specifically on a side comment in the second paragraph. Choice (C) also focuses too specifically on a side comments—the passage is not primarily about disease. Choice (D) is too strong—the passage isn't really promoting any specific action. Choice (E) remains neutral and informational, as does the passage.

2. The case of the Almond Tumbler is most analogous to which of the following? (A) a strain of wheat that can be grown in plentiful quantities but loses much of its nutritional value in the process (B) Arabian horses that are able to run at phenomenal speeds due to centuries of careful breeding designed to enhance those physical attributes (C) vitamins that were purported to provide all of the necessary nutrients but have since been found not to be very effective (D) the dachshund, a popular breed of dog that is nonetheless prone to severe back problems, due to weaknesses exacerbated by targeted breeding (E) the wild rock doves that are most commonly found nesting in the faces of cliffs far from human habitation

Answer: (D) The Almond Tumbler is described as a breed of pigeon that was very popular during the Victorian era. The passage also mentions that the selective breeding used to create that particular kind of bird also led to tiny beaks that kept parent birds from feeding their babies. Therefore, the best analogy would be another animal that is popular even though it has problems due to its design. Choice (A) is incorrect because it leaves out the aspect of popularity. Choice (B) is only positive and you need something that's also negative. Choice (C) is not about something that has been bred for a specific purpose, nor does it deal with popularity. Choice (D) correctly refers to a popular animal with a common health problem. Choice (E) does not refer to pigeons that have been bred by humans.

3. The passage suggests that (A) pigeons were once known for flying with celerity (B) the Almond Tumbler was the most beautiful breed of pigeon (C) Darwin was infatuated with his fancy pigeons (D) modern pigeons are dirtier than the fancy pigeons of yore (E) only scientists should breed new kinds of animals

Answer: (A) For a question this open-ended, it's usually best to check each of the answers against the passage. Choice (A) appears to match the opening line of the passage, which states that the pigeon once stood not only for speed and reliability. Choice (B) goes too far—although many Victorians seems to have loved the Tumbler, there's no evidence that it was definitively the most beautiful. Choice (C) also goes too far—the passage mentions that Darwin was fascinated by his pigeons, not that he was infatuated. Choice (D) draws an incorrect assumption—the passage comments that the common opinion has changed, not the pigeon itself. Choice (E) is not supported by the passage, which states that amateurs, as well as trained individuals, bred pigeons.

Below you'll find GMAT sample questions covering the two question types you'll encounter on the Quantitative section: Problem Solving and Data Sufficiency.

Problem Solving Questions

1. A certain company sells tea in loose leaf and bagged form, and in five flavors: Darjeeling, earl grey, chamomile, peppermint, and orange pekoe. The company packages the tea in boxes that contain either 8 ounces of tea of the same flavor and the same form, or 8 ounces of tea of 4 different flavors and the same form. If the order in which the flavors are packed does not matter, how many different types of packages are possible? (A) 12 (B) 15 (C) 20 (D) 25 (E) 30

Answer: (C) Begin by figuring out how many different ways you can package the tea in boxes that contains 8 ounces of tea, all of the same flavor. There are five flavors, each flavor can come in either loose leaf or bagged form, so 5 flavors x 2 forms = 10 different ways to package the tea in boxes that contain only one flavor each. Now find the number of different ways to package 4 different flavors of the same form per box. In this case, you must choose 4 of 5 possible flavors, and order does not matter, so the formula is 5 x 4 x 3 x 2 ⁄ 4 x 3 x 2 x 1 = 5 different ways to combine the 4 flavors. Each combination can come in either loose leaf for bagged form, so you have 2 different forms x 5 different combinations = 10 total possible ways to combine the 4 flavors in either bagged or loose-leaf form. Thus, the total number of combinations is 10 + 10 = 20 total combinations. The answer is choice (C).

2. Karen sold her house at a loss of 25 percent of the price that she originally paid for the house, and then bought another house at a price of 30 percent less than the price she originally paid for her first house. If she sold the first house for $225,000, what was her net gain, in dollars, for the two transactions? (A) $15,000 (B) $25,000 (C) $60,000 (D) $75,000 (E) $90,000

Answer: (A) If Karen sold her first house for $225,000 and at a loss of 25 percent, then 25 percent of the original price equals $225,000. 75 ⁄ 100 x = 225,000, so x, or the price she originally paid, equals $300,000. Thus, Karen lost $75,000 on the sale of her first house. If she bought a second house for a price of 30 percent less than $300,000, then the second house cost $210,000, so she gained $90,0000. $90,000 - $75,000 = $15,000, so the answer is choice (A).

Sample Data Sufficiency Questions

1. In a certain company, at least 200 people own manual transmission vehicles. If 12 percent of the people who own manual transmission vehicles also own automatic transmission vehicles, do more people own automatic transmission vehicles than own manual transmission vehicles? (1) 5 percent of the people who own an automatic transmissions vehicle also own a manual transmission vehicle. (2) 15 people own both an automatic transmission vehicle and a manual transmission vehicle. (A) Statement (1) ALONE is sufficient, but statement (2) alone is not sufficient. (B) Statement (2) ALONE is sufficient, but statement (1) alone is not sufficient. (C) BOTH statements TOGETHER are sufficient, but NEITHER statement ALONE is sufficient. (D) EACH Statement ALONE is sufficient. (E) Statements (1) and (2) TOGETHER are NOT sufficient to answer the question asked, and additional data are needed.

Answer: (A) According to statement (1), 5 percent of the people who own an automatic transmission vehicle also own a manual transmission vehicle. The question also indicates that 12 percent of the people who own a manual transmission vehicle also own an automatic transmission vehicle. Both figures relate to the total number who own both, so that means that 5 percent of the automatic transmission owners = 12 percent of the manual transmission owners. The overlap in ownership makes up a smaller percent of those who own automatic transmission vehicles, so there must be more people who own automatic transmission vehicles. Statement (1) is sufficient, so you can eliminate choices (B), (C), and (E). Statement (2) indicates that 15 people own both an automatic transmission vehicle and a manual transmission vehicle, so you know that 12 percent of the people who own a manual transmission is equal to 15 people. 12 ⁄ 100 = 15, so x = 125. Thus, there are 125 people who own a manual transmission vehicle. However, you have no further information to allow you to calculate the number of people who own automatic transmission vehicles, so statement (2) is insufficient. The answer is choice (A).

2. What is the value of x ⁄ 2 ? (1) x is 1 ⁄ 5 less than 9 ⁄ 10 (2) x is between 2 ⁄ 5 and 4 ⁄ 5 (A) Statement (1) ALONE is sufficient, but statement (2) alone is not sufficient. (B) Statement (2) ALONE is sufficient, but statement (1) alone is not sufficient. (C) BOTH statements TOGETHER are sufficient, but NEITHER statement ALONE is sufficient. (D) EACH Statement ALONE is sufficient. (E) Statements (1) and (2) TOGETHER are NOT sufficient to answer the question asked, and additional data are needed.

Answer: (A) Statement (1) allows you to find the value of x, so you can answer the question. (If x is 1 ⁄ 5 less than 9 ⁄ 10 , then 9 ⁄ 10 - 1 ⁄ 5 = x. 1 ⁄ 5 = 2 ⁄ 10 , so x equals 9 ⁄ 10 - 2 ⁄ 10 = 7 ⁄ 10 . If x equals 7 ⁄ 10 , then x ⁄ 2 = 7 ⁄ 10 divided by 2, or 7 ⁄ 20 .) Statement (1) is sufficient, so eliminate choices (B), (C), and (E). According to statement (2), x is between 2 ⁄ 5 and 4 ⁄ 5 . That means that one possible value for x is 3 ⁄ 5 , but another possible value is 7 ⁄ 10 . Statement (2) is insufficient, so the answer is choice (A).

Below you'll find examples of how you'll be asked to use a chart, graph, or table to answer questions on the Integrated Reasoning section.

Sample Integrated Reasoning Questions

Item 1: Andre is buying gifts for his office staff. He wants to spend exactly $280 and he can buy either sweatshirts, which cost $22, or baseball caps, which cost $26. In the table below, choose the number of sweatshirts and the number of baseball caps that Andre should buy.

Answer: Sweatshirts, 8; Baseball caps 4 To solve this question, systematically test out the answer choices. The equation you need to solve is 22s + 26h = 280, in which both s and h are integers and s represents the number of sweatshirts and h represents the number of baseball caps. So, start with plugging in 4 for sweatshirts and see if the number of baseball caps is an integer. 22(4) +26h = 280 h = 7.38 Since the number of baseball caps is not an integer, Andre could not have bought 4 sweatshirts. Keep trying more sweatshirts one by one until you find an answer that will you an integer value for baseball caps. 8 sweatshirts will give you 4 baseball caps.

GMAT sample question

Question 2-1 The ratio of the U.S. population in 2000 to the U.S. population in 1900 is closest to __. (A) 1 to 4 (B) 2 to 7 (C) 2 to 1 (D) 3 to 1 (E) 11 to 3

Answer: (E, 11 to 3) According to the graph, the U.S. population in 2000 was a little bit more than 275 million, and the U.S. population in 1900 was a little over 75 million. Since the question asks what the ratio is "closest to," these numbers are good enough to approximate. 275 to 75 can be reduced by 5 to get 55 to 15, which can be reduced by 5 again to get 11 to 3. Alternatively, you could reduce 275 to 75 by 25 to get this same ratio.

Question 2-2 The U.S. population in 1950 was approximately __ of the U.S. population in 1850. (A) 800% (B) 600% (C) 200% (D) 85% (E) 15%

Answer: (B, 600%) The question asks what percent the U.S. population in 1950 is of the U.S. population in 1850. To get this you need to calculate population 1950 ⁄ population 1850 x 100. Since the U.S. population in 1950 is higher, you want something that is greater than 100%. Eliminate 85% and 15%. Since the sentence says "approximate" and also since the remaining answer choices are not close to each other, you can estimate the values. According to the chart, the population in 1950 was about 150 million and the population in 1850 was about 25 million. Therefore, you need to calculate 150 ⁄ 25 x 100 = 6 x 100 = 600%.

Question 2-3 The U.S. population increased by approximately __ from 1900 to 1950. (A) 25% (B) 33% (C) 50% (D) 100% (E) 200%

Answer: (D, 100%) To get percent increase, you need to use the formula difference ⁄ original x 100. The population in 1900 was about 75 million, and the population in 1950 was about 150 million. The difference between the two figures is 75 million. Therefore, the percent increase is 75 ⁄ 75 x 100 = 100%.

Below you'll find a sample Analytical Writing Assessment (AWA) question. On the GMAT you'll have 30 minutes to write a critique of the argument.

Analysis of an Argument

The following appeared as part of a medical advertisement in a magazine.

A new medical test that allows the early detection of a particular disease will prevent the deaths of people all over the world who would otherwise die from the disease. The test has been extremely effective in allowing doctors to diagnose the disease six months to a year before it would have been spotted by conventional means.

Discuss how logically convincing you find this argument. In explaining your point of view, be sure to evaluate the line of reasoning and the use of evidence in the argument. For example, it may be necessary to consider what questionable assumptions underlie the thinking and what other explanations or counterexamples might weaken the arguments conclusion. You can also discuss what kind of evidence would strengthen or refute the argument, what changes in the argument would make it more logically persuasive, and what, if anything, would enable you to better evaluate its conclusion.

How will you score?

Take a GMAT practice test with us under the same conditions as the real thing. You'll get a personalized score report highlighting your strengths and areas of improvement.

START A FREE PRACTICE TEST

  • GMAT  

Featured Business Schools For You

Find MBA Programs Matched to Your Interests

Explore our featured business schools to find those that are looking for students like you.

Best Online MBA seal

Top Online MBA Programs

On a mission to increase your salary? Our Top 50 Online MBA ranking is based on academics, career outcomes, tech platforms, and more.

Best Career Prospects

Best Career Prospects

Find out which schools have the best track records for getting students jobs—and the highest starting salaries.

Top Entrepreneurship 2023 seal

Top Schools for Entrepreneurship

Ready to build your own business from the ground up? Check out these 50 graduate programs.

gmat essay answer examples

Free MCAT Practice Test

I already know my score.

gmat essay answer examples

MCAT Self-Paced 14-Day Free Trial

gmat essay answer examples

Enrollment Advisor

1-800-2REVIEW (800-273-8439) ext. 1

1-877-LEARN-30

Mon-Fri 9AM-10PM ET

Sat-Sun 9AM-8PM ET

Student Support

1-800-2REVIEW (800-273-8439) ext. 2

Mon-Fri 9AM-9PM ET

Sat-Sun 8:30AM-5PM ET

Partnerships

  • Teach or Tutor for Us

College Readiness

International

Advertising

Affiliate/Other

  • Enrollment Terms & Conditions
  • Accessibility
  • Cigna Medical Transparency in Coverage

Register Book

Local Offices: Mon-Fri 9AM-6PM

  • SAT Subject Tests

Academic Subjects

  • Social Studies

Find the Right College

  • College Rankings
  • College Advice
  • Applying to College
  • Financial Aid

School & District Partnerships

  • Professional Development
  • Advice Articles
  • Private Tutoring
  • Mobile Apps
  • International Offices
  • Work for Us
  • Affiliate Program
  • Partner with Us
  • Advertise with Us
  • International Partnerships
  • Our Guarantees
  • Accessibility – Canada

Privacy Policy | CA Privacy Notice | Do Not Sell or Share My Personal Information | Your Opt-Out Rights | Terms of Use | Site Map

©2024 TPR Education IP Holdings, LLC. All Rights Reserved. The Princeton Review is not affiliated with Princeton University

TPR Education, LLC (doing business as “The Princeton Review”) is controlled by Primavera Holdings Limited, a firm owned by Chinese nationals with a principal place of business in Hong Kong, China.

gmat essay answer examples

  • GMAT Eligibility Criteria
  • GMAT Registration
  • GMAT Online Classes
  • GMAT Verbal Syllabus
  • GMAT AWA Syllabus
  • GMAT Integrated Reasoning
  • GMAT Quantitative
  • GMAT Sample Papers
  • GMAT Sample Questions
  • GMAT Question Papers
  • GMAT Practice Test
  • GMAT Verbal Questions
  • GMAT Quant Questions
  • GMAT Coaching
  • GMAT Mock Test
  • GMAT Webinar

Home » Free GMAT Prep » GMAT Essay Template

Experience (Optional)

GMAT Essay Template

Are you preparing for the GMAT AWA section? Are you not familiar with the GMAT essay template? Worry not! We will help you acquaint yourself with the GMAT essay template and how it can help you when attempting the AWA section.

The Analytical Writing Assessment (AWA) is the only section in the GMAT that requires you to draft an essay, which is why it is also known as the “essay section”. Before we begin, let us take a quick look at the GMAT AWA section. 

GMAT AWA  

The GMAT Analytical Writing Assessment tests your ability to critically analyse an argument. Once you have critiqued the argument, you need to articulate your analysis in the form of a coherent, logically sound essay in standard written English.  The AWA section consists of a task — Analysis of Argument, which you have to complete within 30 minutes. The Analysis of Argument requires you to analyse if the argument is logically sound or not. To do so, you must analyse the reasoning and the evidence used in the given argument. Hence, attempting the AWA section requires you to brush up on your critical thinking and logical reasoning skills. Since you have only 30 minutes to complete this writing task, you need to plan how to draft your essay. Having a GMAT essay template in hand will help you plan how you want your AWA essay to look. Let us have a look at how a GMAT essay template helps you with the same. 

GMAT Essay Template: How it Helps?

The GMAT essay template refers to a rough outline of your AWA essay. Since you have to complete the writing task within a limited time frame, having a template in hand will help you draft an effective essay. The GMAT essay template also serves as a blueprint for your actual AWA essay. With a readymade blueprint in hand, you can approach the writing task with much ease. Additionally, an essay template helps you jot down all the main ideas of your essay before you begin your essay. Putting down your main ideas helps you organise them better and ensures your trail of thought isn’t disrupted. All in all, a GMAT essay template is a tool that ensures that you have drafted an effective and coherent essay. 

GMAT Essay Template: A Sample Template  

Most AWA essays that are scored between 5 to 6, have followed a similar template. Keeping this in mind, let us take a look at what an ideal GMAT essay template should contain. The ideal or effective GMAT essay template should contain the following: 

  • Structure of the essay: The template must entail how you are going to structure your essay and should follow a paragraph-by-paragraph format. 
  • Content: The template must mention the ideas or points that each paragraph must contain. Ideally, each paragraph should be limited to one main idea. 
  • Pre-written phrases: The template must also contain certain phrases, keywords or sentence stems that you are planning to use in the paragraphs. You can use them to make a note of the flaws you have stumbled upon in the argument. 

Now, let us take a look at a sample GMAT essay template. The sample GMAT essay template is as follows: 

  • Introductory Paragraph: The introduction should not be longer than two to three lines. You have to quickly summarise the argument in your introduction and state your opinion on the same. Additionally, you can choose to state that the author’s argument is not all that bad. 
  • Body Paragraph: The essay can have around two to three body paragraphs. Each paragraph must discuss one flaw that you have recognised in the argument. You need to support the claims you make with proper evidence. You can also explain how the flaw undermines the argument. 
  • Conclusion Paragraph: You must conclude your essay by stating that the argument is flawed. You must also give a brief recap of your analysis and support it with a concluding statement. Additionally, you can also specify what the author could have done to improve their case and suggest ways for them to rectify the flaws in the argument. 

The idea behind the GMAT essay template is to assist you in drafting an effective AWA essay. While this is a sample essay template, you can always use your own essay template. You can attempt a few AWA writing prompts and formulate your essay template to understand which one works for you. Additionally, you can go through various sample essays with high scores and mimic the essay template used in them as well. 

We hope to have acquainted you with the GMAT essay template. Now that you are aware of how to use an essay template to draft an effective AWA essay, what are you waiting for? Start practising today and ace the AWA section.

All the best!

gmat essay answer examples

Leave a Reply Cancel Reply

Your email address will not be published. Required fields are marked *

Save my name, email, and website in this browser for the next time I comment.

gmat essay answer examples

Before Leaving

Thank you for your message. it has been sent..

gmat essay answer examples

Our Offerings

For students, for working professionals, profile builder, profile booster, #teamcareerlabs, leaders who inspire, our journey so far, terms & conditions, privacy policy.

Important GMAT Topics

Important gre topics.

gmat essay answer examples

How to Write a Powerful MBA Essay—With Examples

The MBA essay is critical to your business school application. Read our guide to writing the perfect MBA essay, with successful admit examples.

Posted April 4, 2024

gmat essay answer examples

MBA Week (June 3-6)

Starting monday, june 3.

10:00 PM UTC · 60 minutes

Table of Contents

What is the mba essay.

The MBA admissions essay.

Those words alone are enough to make most MBA candidates run screaming. Writing in general is hard enough. Writing about why you want an MBA? Your short-term goals and career aspirations? What matters to you most, and why? Forget it.

Of course, you still have to write these essays.

The MBA essay is perhaps  the most important part of the business school application. (It's also getting more and more important by the day, with some business schools moving away from traditional, quantitative measuring sticks, like the GMAT and the GRE.) Every other part of the application — your GPA, your test scores, your letters of recommendation — are quantified, cut and dried, or out of your control. The essay is your chance to show up as a fully realized MBA candidate, with hopes, dreams, and vulnerabilities. Admissions committees are not simply assessing your candidacy as a future leader — they're looking to admit human beings. That's where the MBA applicant essays come in.

That being the case, rather than being intimidated by it, treat the essay like the opportunity that it is — the chance for you to highlight your unique, iridescent self; the only moment in the MBA admissions process (prior to the interview) when you can speak directly to the admissions officers; the time when you'll show them who you really are. It's not easy to write something that will do that, of course, but with the tips and tricks in this guide, and some help from one of Leland's vetted, world-class admissions coaches, we know you can do it. Give the essay the time, attention, and respect it deserves, and you'll be on your way to an offer of admission at your dream school.

Without further ado, let's dive in!

gmat essay answer examples

Ultimate MBA Essay Guide

See the MBA essay prompts, top tips from experts, and real examples from admits with this comprehensive guide.

How Long Will My MBA Essay Take?

First thing's first: let's talk about timing.

The MBA application is a behemoth; between exams, resumes, gathering your official transcripts, letters of recommendation, and the applications themselves, there's a lot to juggle. That being the case, we suggest you give yourself ample time to draft, write, and revise your essays. The last thing you want is to be rushed to the finish line.

So, give yourself  at least three months to write your MBA essays. That should allow you ample time to draft, write, and edit. For more information on timing your entire b-school application, click here for  A Comprehensive MBA Application Timeline--With Chart .

Now, on to the critical question:

Free trial!

undefined's profile

From 110 top coaches

Access a library of videos, templates, and examples curated by Leland’s top coaches.

Example essays.

Example Essays Image

Example Resumes

Example Resumes Image

Application Prep

Application Prep Image

Video Courses

Video Courses Image

What Makes a Great MBA Essay?

At the highest level, the answer is the one that is truest to you. The whole point of these essays is to shine through as an authentic, vibrant human being, so the best essays are the ones that cut through the clutter, and allow you do to that.

Which begs the question — how  do you cut through the clutter and shine through as a vibrant human being? Here are four critical tips to follow as you begin thinking about your essays.

1. Answer the Question

This one sounds obvious, but you'd be surprised how many applicants launch into their story, get carried away, and forget to answer the question. Follow the prompt, and answer the question the admissions committee has asked you. Those prompts can actually be very useful when writing your essays — it's a great deal harder to write when you have no guidance or guardrails. With the MBA essays, you have a very specific question you need to answer. So answer it!

2. Be Specific

Another mistake some MBA applicants make is to stay at a high level in their essays, keeping their writing abstract and therefore inaccessible to the admissions committee. If at any point, an admissions officer could replace your name with the name of another applicant, then your essay isn't getting deep enough. It's not enough, for instance, to say that you suffered adversity in high school, or that you really, really want a Wharton MBA. You need to explain, in detail, the adversity you faced, and give concrete and unique reasons why you think Wharton is the right program for you. The best essays offer hyper-specific examples and anecdotes, with details and anecdotes that no other candidate could bring to the table. To get those anecdotes, we recommend using the STAR template, as explained below:

  • Situation : What was the situation you were facing? Where were you? How old were you? If you were in a professional role during this anecdote, what was the role, and how long had you been in it? If you were volunteering, at what organization? How long had you been volunteering there? Why did you start? Offer all the relevant information that the admissions readers will need to understand your story.
  • Task : What was the task at hand? What went wrong? In your professional role, what was the challenge you faced? In that volunteering experience, what were the hurdles you had to overcome? You can't have a good story without conflict or tension, so after you set up the anecdote, explain what that conflict or tension was (and remember, be specific!).
  • Action : What was the action you took to resolve the problem? What did you have to do to fix that issue at work? How did you clear that hurdle in your volunteer experience? Again, be specific about how you came through on the other side of that conflict/tension — and while you're doing it, highlight your leadership capabilities as much as possible! Remember that top MBA programs are looking for future leaders who can assess a situation and decisively take action. (We'll say a bit more about this below, in the Personal Statement section.
  • Result : What was the result of your action? If you were facing a growth problem at work, were you able to increase sales? If so, by what percentage? If you were advocating for diversity and inclusion at your local charity, what new programs did you implement to help with that effort, and what was the enrollment like in those new programs? Detail what happened in your anecdote with as much specificity as possible — and quantify, quantify, quantify!

3. Get Vulnerable

Most MBA admissions essay prompts are written with the goal of getting to know as much about you as possible in the shortest number of words. To do that, you're going to have to share real things from your life — to get personal, intimate, and vulnerable. Do not shy away from this. If you're starting to get emotional during the reflection, drafting, and writing process, good — that means you're on the right track. Keep going. Pro tip: If it’s making you cry, it will make them cry. Another good rule of thumb is to put something real and true on the table. Admissions officers have to read literally thousands of applications from thoroughly qualified individuals, some of whom might come from similar roles to yours, with letters of recommendation from equally impressive supervisors. In order to cut through that noise, you'll have to share something honest. If you're doing it right, this can feel risky. At some point, you’ll likely think to yourself: “Can I say that?” The answer is: “Yes.” Of course, there is a line, you don’t want to be crass or offensive but err on the side of being open and authentic. The very worst thing you can do is be overly cautious, and write something you think will please the admissions committee. These poor people have to read thousands of essays. If yours is just like everyone else’s, they’ll fall asleep. Don’t let that happen. Wake them up by putting yourself —your true, bright, vibrant, quirky self—on the page.

4. Don't Exaggerate

Finally, do not exaggerate, over-inflate, or lie. This goes without saying, but admissions committees are looking for honest candidates. The surest way to get rejected is to lie about something. (Business schools do a background check on you before you're properly admitted, so they will find out.) Don't be the person who over-inflates on their essays, then has their offer letter rescinded.

The Types of MBA Essays

All right — since we've covered high-level approaches to the MBA essays, it's time to dig into the various types.

There are three general categories of MBA essays you'll see across the board.

1. Personal Statement

These questions ask you to offer up something sincere about yourself. They'll often touch on such things as your values and your character. In these, you'll want to be as authentic as possible, while also highlighting attributes like leadership, intellectual vitality, and teamwork, that business schools are looking for. Here are a few examples of personal statement essays:

  • As we review your application, what more would you like us to know as we consider your candidacy for the Harvard Business School MBA program? (HBS)
  • What matters most to you, and why? (Stanford GSB)

2. Why an MBA/Why This School

The next category of essays is the "Why an MBA" / "Why This School" set.

In these, schools first want to hear about how an MBA will fit into your career, both short and long term. Top MBA programs are looking for candidates who will: first of all, be gainfully employed upon graduating, second of all, have an illustrious career that will make their institution look good and encourage future generations of applicants to apply, and third, be consistent and generous donors. That being the case, they want to know about your career trajectory, and how an MBA will fit into it.

Pro tip: Here, you want to be ambitious and inspiring in laying out your future career, but not naïve. Walk the line between shooting for the stars and sounding dreamlike and uninformed.

In this set of questions, you'll also encounter questions geared at figuring out why you would want to attend a specific school. MBA programs want to know that you're serious about attending their school — yield, or the percentage of admitted candidates who accept their offers of admission, is an important metric for them — but they also want to envision how you'll contribute to their admitted class. What will you uniquely bring to the table, the things that you'll do that the other candidates wouldn’t be able to offer?

We've heard former deans of business schools say that, in choosing a class, they're curating a world-class dinner party, and that each person invited to the dinner party has to bring something different. What will you bring to the dinner party?

Pro tip: To demonstrate that you've done your research, and to help the admissions committee envision you in their program, indicate which classes you might take when earning your MBA and why, which professors you might hope to study with, and in which clubs you might participate.

Here are a few examples of "why MBA / why this school" essays:

  • How is a Columbia MBA going to help you? (Columbia)
  • What do you hope to gain professionally from the Wharton MBA? (Wharton)
  • Why Stanford? Describe your aspirations and how your Stanford GSB experience will help you realize them. (Stanford GSB)

3. Behavioral/Other

Finally, most other essays will either be behavioral, asking you about experiences, traits, strengths, weaknesses, and achievements. There's a wide variety of topics here, but all the guidelines from above apply, with the final note to always prioritize authenticity (as mentioned in the Personal Statement section) and leadership ability (remember, business schools are choosing future leaders). Here are a few examples of behavioral/other essays:

  • Describe the biggest commitment you have ever made. (Yale SOM)
  • Tell us about your favorite book, movie, or song and why it resonates with you. (Columbia)
  • Think about times you’ve created a positive impact, whether in professional, extracurricular, academic, or other settings. What was your impact? What made it significant to you or to others? (Stanford GSB)

Top MBA Program Essay Prompts (Updated 2022)

To help you get started, we've compiled the required prompts from a few top MBA programs below:

1. Harvard Business School (HBS)

As we review your application, what more would you like us to know as we consider your candidacy for the Harvard Business School MBA program? (900 words)

For more information, visit A Guide to the HBS Essay .

2. Stanford Graduate School of Business

What matters to you most, and why? (650 words)

Why Stanford? (400 words)

Read What Matters Most When Writing the GSB Essays.

How do you plan to use the Wharton MBA program to help you achieve your future professional goals? You might consider your past experience, short and long-term goals, and resources available at Wharton. (500 words)

Taking into consideration your background – personal, professional, and/or academic – how do you plan to make specific, meaningful contributions to the Wharton community? (400 words)

For Wharton-specific advice, visit A Guide to the Wharton Essays .

4. Columbia Business School

Essay 1: Through your resume and recommendation, we have a clear sense of your professional path to date. What are your career goals over the next three to five years and what, in your imagination, would be your long-term dream job? (500 words)

Essay 2: The Phillips Pathway for Inclusive Leadership (PPIL) is a new co-curricular program designed to ensure that every CBS student develops the skills to become an ethical and inclusive leader. Through PPIL, students attend programming focused on five essential diversity, equity, and inclusion skills: Creating an Inclusive Environment, Mitigating Bias, Communicating Across Identities, Addressing Systemic Inequity, and Managing Difficult Conversations. Tell us about a time you were challenged around one of these five skills. Describe the situation, the actions you took, and the outcome. (250 words)

Essay 3: We believe Columbia Business School is a special place. CBS proudly fosters a collaborative learning environment through curricular experiences like our clusters and learning teams , an extremely active co-curricular and student life environment, and career mentorship opportunities like our Executives-in-Residence program .Why do you feel Columbia Business School is a good fit for you academically, culturally, and professionally? Please be specific. (250 words)

5. Chicago Booth

How will the Booth MBA help you achieve your immediate and long-term post-MBA career goals? (250-word minimum)

An MBA is as much about personal growth as it is about professional development. In addition to sharing your experience and goals in terms of your career, we’d like to learn more about you outside of the office. Use this opportunity to tell us something about who you are… (250-word minimum)

Read more at A Guide to the Booth Essays .

6. Kellogg Northwestern

Kellogg’s purpose is to educate, equip and inspire brave leaders who create lasting value. Provide a recent example where you have demonstrated leadership and created value. What challenges did you face and what did you learn? (450 words)

Values are what guide you in your life and work. What values are important to you and how have they influenced you? (450 words)

Read How to Nail Your Kellogg MBA Application Essays

7. MIT Sloan

MIT Sloan seeks students whose personal characteristics demonstrate that they will make the most of the incredible opportunities at MIT, both academic and non-academic. We are on a quest to find those whose presence will enhance the experience of other students. We seek thoughtful leaders with exceptional intellectual abilities and the drive and determination to put their stamp on the world. We welcome people who are independent, authentic, and fearlessly creative — true doers. We want people who can redefine solutions to conventional problems, and strive to preempt unconventional dilemmas with cutting-edge ideas. We demand integrity, respect, and passion.

Taking the above into consideration, please submit a cover letter seeking a place in the MIT Sloan MBA program. Your letter should conform to standard business correspondence, include one or more professional examples that illustrate why you meet the desired criteria above, and be addressed to the Admissions Committee (300 words or fewer, excluding address and salutation)

Applicants are required to upload a 1 minute (60 seconds) video as part of their application. In your video, you should introduce yourself to your future classmates, tell us about your past experiences, and touch on why MIT Sloan is the best place for you to pursue your degree.

How to Start Your MBA Essay

So you've read about the types of essays, and seen some of the prompts from top MBA programs. Now it's time to actually start diving into the essay.

The very first thing to do, before putting pen to paper, is to look inward.

Why do  you want an MBA? What role will this degree play in your professional growth? How do you imagine it will shape your life? What do you want out of your career? What is the most important thing in the world to you?

Yes, these are life’s deep-end questions, but you’ll need to tackle them in these essays, so before you start writing, take the time to think through them. Go for a run, swim some laps, bake a cake—however you get into the flow — and start a dialogue with yourself. Put down your work, turn your phone off, and give your mind permission to go to the places it usually avoids. That’s a good place to start. That’s where the answers are.

Pro tip: The first sentence is the hardest one to write. When you're starting out if it can intimidating and anxiety-producing. The trick is to simply put  anything  down — and don't look back. Keep putting one sentence after the other. You can edit later: let whatever comes to you out onto the page. If you’re struggling with self-critique, dim your computer screen until you can’t even see the words you’re typing. Then keep going.

Additional Tips & Tricks

Once you've started your essay, it's a matter of persistence: keep writing, then keep drafting and editing until you have something you're really proud of.

To help you with that process, here are a few more tips and tricks:

  • Take Breaks

When you hit the wall — you will hit the wall — stop. This is your brain telling you it needs to do something else. Walk your dog. Take a lap around your room. Eat some cheese. Your body needs sleep every night to function; your mind is the same way. That next leap of inspiration will come exactly at the moment when you’re least expecting it.

  • Read it Out Loud

When you finally have a draft, print it and read it out loud to yourself. Your ear will catch things your eyes miss. Reading out loud is the best way to pick up on spelling errors, clunky transitions, and paragraphs that still need ironing out. It’s also a good way to envision how the admissions committee will experience your essay.

Don’t be precious with your essay. Send it to anyone willing to read it. Solicit as much feedback as you can. If you don’t like what people have to say, you don’t have to incorporate it, but you need an impartial third party to give notes on what they’re seeing, thinking, and feeling. (You’re too close to things to do it for yourself.) This is where a Leland coach comes very much in handy!

  • Complete Everything Early

This is more of a timing consideration, but you do not want to trip at the finish line because your internet went down the night before the deadline, or your credit card was denied when paying your application fee (it's happened before). Don't let that be you!

Here is another article to get you started, written by an expert essays coach: 7 MBA Essay Tips to Make You Stand Out in 2022 .

Example MBA Essays

Finally, here are two essays to help inspire you. The first, a personal statement essay, was submitted by an admit to Berkeley Haas' Executive MBA program; the second, a career goals / why MBA essay, was submitted by an admit to Chicago Booth's deferred MBA program.

Haas Admit:

A person’s identity is shaped by many different aspects, including family, culture, personal interests, and surrounding environments. Please share a facet of your identity or story that is essential to who you are. (300 words) My upbringing in India, filled with countless myths and legends, had a profound influence on me. The most formative tale was about a sage who prays for years to the goddess of knowledge, but in vain. In the end, the goddess didn’t appear for the sage because he was turning his prayer beads the wrong way! As a child, this story upset me: the sage worked so hard and had the right intentions. As an adult, though, I’ve come to realize that the goddess of knowledge was right: you can’t succeed unless you do things the right way. Seven years ago, two friends and I started a company, XXXX: a digital health platform that would allow patients to store medical records online and consult doctors remotely. We had early success—we brought on 2,000 patients at XXXX, a gynecology clinic in XXXX—but ultimately we didn’t have the resources to properly scale, and had to shut the company down. Among the many lessons I learned, the most valuable was that ideas and hard work are common; businesses succeed or fail based on execution—on doing things the right way. Two years ago, I relearned this lesson in the most painful way possible: when my marriage ended. My wife and I loved each other, but we weren’t there for each other when it mattered most. Our feelings weren’t enough—we had to back them up with the right actions. It’s disheartening when you have good intentions but still fall short. When this happens, though, you have to keep trying—because eventually you will do things the right way. I carry the story of the sage with me always, not as a harsh lesson, but as a motivating goal: one that keeps me striving towards doing things the right way.

Booth Admit:

How will the Booth MBA help you achieve your immediate and long-term post-MBA career goals? (250 word minimum)
I want to start a geothermal company that will help lead the energy transition away from fossil fuels and toward renewable energy—by targeting existing oil wells as sites for geothermal plants. Oil fields are close to electric grids and have high nearby subsurface temperatures, making them ideal sites for geothermal plants. By building geothermal infrastructure nearby, my company will produce cleaner, cheaper energy, making it more profitable for operators to switch from oil to geothermal. As oil companies decommission their wells, I’ll negotiate for their land rights, so I can use their existing wells for new geothermal vents. I want my company to prove the case for economically viable, carbon neutral energy production. After getting an MBA I want to start a geothermal company which will help me lead the energy transition away from fossil fuels to renewable energy. I plan to target developed oil fields in Texas, where, in many places, producing wells are flowing enough hot fluid to generate clean energy. Using this geothermal heat, the carbon footprint of oil and gas extraction will decrease as fewer fossil fuels are utilized to power surrounding infrastructure. As the wells approach their economic life, I will negotiate the lease from various operators, saving them millions in plug and abandonment costs, and retrofit the wells for direct geothermal energy production via closed loop binary fluid systems, bringing emissions to zero. To accomplish this goal, I need to shore up my knowledge of energy economics and entrepreneurial finance, develop a strong sense of leadership, and build a network of like minded individuals that will help me lead the transition and I believe I can get those things at Chicago Booth. My immediate career goal is to develop my first co-production site in Shelby County, Texas at the Blanton well site, which produces abnormally heated fluid from the flanks of an active salt dome. Before investing in capital expenditures, developing a strong sense of energy economics and broader markets is necessary to verify financial feasibility. University of Chicago, through the Graduate-Student-At-Large: Business program, is already allowing me to accomplish this goal with my enrollment in “Microeconomics” with Professor Andrew McClellan. His instruction helped me understand the impact taxes and subsidies have on market equilibrium, an important aspect of renewable energy as green energy tax incentives continue to change on a yearly basis. As my company continues to grow, having a strong finance and accounting foundation is imperative to building and sustaining a healthy company. Electives such as “Accounting for Entrepreneurship: From Start-Up through IPO” will provide the skills I need to be successful by following the life-cycle of a business that originates as a start-up, and covers topics such as building an initial accounting infrastructure. I understand that execution of the business is as important as developing the idea and proof of concept, and Booth is the best place for me to develop financial fluency. Leading the energy transition will require a strong sense of leadership. Not only will I need to lead those I get to work with over my career, but to lead the energy transition, and reverse the impact fossil fuels have had thus far, I must have the emotional intelligence to inspire others to join me in my journey. The “Interpersonal Dynamics” course at Booth will allow me to develop my communication skills and better understand the emotions and perceptions of my colleagues. These skills, synthesized with leadership development acquired in “Leadership Practicum” will prepare me to act as a relational leader, who understands the needs of others. As a relational leader I hope to foster an environment which promotes happiness, and maximizes efficiency, not only to make our efforts in changing the world more successful, but to excite other people to join our cause.
To find the greatest chance of success in leading the energy transition, I will need a network of like-minded individuals who can provide a diversity of thought. Chicago Booth provides the opportunity to develop that network through different community experiences. The Energy Club’s “Energy Forward” conference, which designates time to topics in oil and gas and renewable energy will allow me to hear from industry leaders, build meaningful relationships with peers and contribute my sector experience to the public forum as I learn from those around me. Opportunities through the Entrepreneurship and Venture Capital Group such as “SeedCon” will help me connect with successful entrepreneurs and early-stage investors whose ideas and funding might change the course of my venture’s trajectory. Even in the GSALB program I have had the opportunity to connect with other students in various sectors, including the energy industry. I hope to continue to strengthen those connections and continue building new ones with matriculation into the full time program.

Here are several other articles that you may find helpful as you put together your MBA application:

  • The Most Frequently Asked Questions on MBA Applications
  • How to Answer the "Why an MBA?" Essay Question
  • My Top Piece of Advice for MBA Applicants
  • How I Nailed My MBA Interview and Gained Admission to Top 10 Business Schools
  • 4 Expert Tips on Paying for Business School

Browse hundreds of expert coaches

Leland coaches have helped thousands of people achieve their goals. A dedicated mentor can make all the difference.

Browse Related Articles

gmat essay answer examples

January 9, 2024

How to Nail Your Kellogg MBA Application Essays

Tips and tricks that will help you craft the best application essays possible and get admitted into Kellogg.

gmat essay answer examples

January 28, 2024

How to Nail Your Stanford GSB Short Answer Questions

Learn how to stand out with your Stanford GSB short answer questions.

gmat essay answer examples

January 4, 2024

HBS 2+2 Deferred MBA Essay Prompts & Tips (2024)

As of 2024, HBS has changed its deferred MBA essay prompts away from the traditional, "What else should we know about you?" to three smaller essays. Read more and nail your HBS 2+2 application here.

gmat essay answer examples

May 10, 2024

How to Ace Your HBS MBA Interview

Interviewing for the deferred program at Harvard Business School? Ace your interview with these helpful tips and tricks, including sample questions from actual interviews—only on Leland.

gmat essay answer examples

May 11, 2024

An Overview of the HBS 2+2 Program—and How to Kick Off Your Application

HBS 2+2 is one of the most prestigious deferred MBA programs in the country. Here's an overview, with some tips on how to start your application.

gmat essay answer examples

May 9, 2024

How to Get the Perfect MBA Letter of Recommendation—With Examples

The ultimate guide to the MBA recommendation letter, including examples of letters that helped applicants earn admission to top 10 MBA programs.

gmat essay answer examples

March 12, 2024

MBA Essay Too Long? Here's How to Fix It

The word count of most MBA application essays is very limited. This article by an expert Leland MBA coach will give you 8 tips to help you keep your essays under the limit while still maintaining a compelling story.

gmat essay answer examples

January 31, 2024

How to Nail the "Why Wharton" MBA Essay

Learn how to craft a compelling 'Why Wharton' MBA essay that sets you apart from the competition. Elevate your application to stand out.

gmat essay answer examples

May 4, 2023

Why ChatGPT Can’t Write Your Personal Statement

While ChatGPT is multifaceted, there is a compelling argument against using AI for your personal statements. Here is one expert's take on the revolutionary technology and application essays.

gmat essay answer examples

Top 10 Deferred MBA Programs in the US—and How to Get In (2024)

Aiming for an MBA, even while you're still in college? Perfect—learn all about deferred admission MBA programs and receive key insights into the DMBA application process.

gmat essay answer examples

Craft a Powerful Essay for Stanford GSB: What Matters Most & Why?

A GSB MBA, expert admissions coach, and pro writer, outlines his top advice for nailing the challenging and broad Stanford essay prompt, to help you get into one of the most prestigious MBA programs in the world.

gmat essay answer examples

A Guide to the Columbia Business School Essays (2023-2024)

Coach Melanie E. walks you through each Columbia Business School essay prompt for the 2023-2024 cycle, breaking down what adcoms are looking for and offering expert advice on how to nail your responses.

close

Customized for You

Track Your Progress

Practice Pays

Practice thousands of GMAT questions with top expert solutions.

Identify and improve upon mistakes efficiently using our Error Log.

Get the latest tips and news from our top GMAT professionals.

- it’s free and easy!

Thank you for using the timer! We noticed you are actually not timing your practice. Click the START button first next time you use the timer. There are many benefits to timing your practice , including:

We’ll give you an estimate of your score

We’ll provide personalized question recommendations

Your score will improve and your results will be more realistic

gmat essay answer examples

Master Data Sufficiency Questions on Data Insights (Free Webinar)

Master Hard MSR Questions on GMAT Focus (Free Webinar)

Conquer Algebra on the GMAT Focus Edition

GMAT Online Course at $159 | 2X more Comprehensive Content for GMAT Focus Edition

FREE Focus Edition Mock!

MBA Spotlight Fair

gmat essay answer examples

05:30 AM PDT

07:30 AM PDT

gmat essay answer examples

11:00 AM IST

01:00 PM IST

01:30 PM IST

11:30 PM IST

gmat essay answer examples

08:00 PM PDT

09:00 PM PDT

gmat essay answer examples

12:00 PM PDT

Sample AWA Prompt and 6/6 Essay

User avatar

The post is bookmarked successfully

Enjoy this post? Rate it!

GMAT essay topics

GMAT essay topics | 3 examples included to ace the exam!

The gmat essay topics are distinct from most essays you’ve written in college. it involves critical thinking skills, how well you articulate the matter without picking a side, and so on. read on to know more, table of contents, all you need to know about the gmat essay section, gmat essay topics | things to keep in mind, example 1: it is an excerpt from the grocery store’s business plan, example 2: the publisher of the mercury, a weekly newspaper, issued the following announcement, example 3: protesters at waymarsh state college, #1. time your gmat essay practice, #2. write your gmat essays, #3. assess the quality of your gmat essays, key takeaways.

The GMAT essay section provides a one-paragraph prompt containing a specific argument. This document includes practice questions to help you improve your essay section , help you analyze how well you have reasoned the statement, and so on. While attempting the essay section, you have to keep certain aspects in mind. To help you nail the exam, here are some GMAT essay topics that you must practice positively. 

Before proceeding ahead with the GMAT essay topics, here are a few things you need to keep in mind –

  • Ensure to analyze the argument’s logic and the incorporation of proof in the assertion during your discussion.
  • Consider the debatable assumptions that underpin thoughts and the possible theories or counterpoints that might undermine the conclusion. 
  • You can also talk about what kind of evidence would help or hurt the argument. 
  • Additionally, consider what changes to the debate would make it more logical and what would help you help assess its outcome.

GMAT essay topics |Examples that you must know of

Now that we’ve covered what the essay task requires, let’s first look at some sample GMAT essay topics from the official list.

No clear categories of provokes affect your assessment , but for a representative sample. It’s worth noting that the construction of each assertion does not fall within these boundaries; political prompts can employ the same flawed confrontational strategies as business or health and science prompts, for example.

Practicing GMAT essay topics before the actual exam

‘Even though we introduced a pharmacy segment to our supermarket this year, our overall sales have risen by 20%. The main concern of the customer is the convenience provided by one-stop shopping. Adding a clothing department and a vehicle supplies and repair shop is the most certain way to boost our profits in the coming years. In the coming years, we must also plan to add different departments and assistance, including a restaurant and a landscape shop. We will have a competitive edge over other local retailers since we are the only store in the district that offers such a wide range of services.’

Start writing answers wherein you discuss one or more clashing framework explanations and explain how the summaries can compensate for the proof stated in the proposition. Finally, look for logical flaws in the ending, facts, and rationale.

GMAT essay topics | 3 examples included to ace the exam!

‘Since the launch of a competing lower-priced newspaper, The Bugle, five years ago, The Mercury’s circulation has dropped by 10,000 readers.” The best way to encourage more people to read The Mercury is to lower its price less than that of the Bugle until circulation returns to pre-recession levels. Mercury’s increased circulation will entice more businesses to purchase ad space in the paper.’

Attempt writing responses in which you deliberate one or more contending framework explanations and describe how your descriptions can account for the evidence stated in the argument. Finally, examine the conclusion, evidence, and reasoning for logical flaws.

The following appeared in a local newspaper’s editorial section:

‘This past winter, 200 Waymarsh State College students marched to the state’s capital to hold rallies against proposed budget cuts to various state college programs.  On the other hand, the other 12,000 Waymarsh students either remained on campus or left for the winter holidays, indicating that they were less concerned with their education. Because the non-protesters outnumber the protesters, they represent the state’s college students more. As a result, the state legislature should disregard the protesting students’ pleas.’

Start to write solutions in which you clarify a few conflicting arrangement reasons and illustrate how the overviews can make up the difference for the verification outlined in the hypothesis. Finally, take a glance for technical inaccuracies in the conclusion, factual information, and reasoning skills.

GMAT essay topics | Tips to ace the section!

The tips that follow will assist you in preparing for any GMAT essay topics you may encounter on test day –

When practicing GMAT essay topics, remember that you have a strict 30-minute time limit for your essay.

You should start writing all of your practice essays on a compute r to simulate the situations of the computer-based GMAT.

After you’ve completed your practice GMAT essays, try to grade them using the 6-point grading set of guidelines and by correlating your writing to the GMAC’s sample essay.

  • The GMAT essay is unlike most of the essays you’ve written in college. 
  • The GMAC provides a list of the most formal GMAT essay topics asked in the exam. 
  • Ensure to practice the essay topics every day. 
  • The more you practice, the better you get at it. 
  • Answer mock GMAT essays. It will help you grasp the technique of writing excellent essays. 
  • Ensure that you articulate the matter well. Use your critical thinking skills for the same. 

If you need assistance in your GMAT preparation, we are here for you. Get in touch with our experts today!

Liked this blog? Then read the blog on the GMAT score chart | 5 points to enhance your exam strategy!

Question 1: What happens if a student fails the GMAT?

Answer: A student can take the GMAT up to five times yearly. As a result, even if you fail once, you will be given another chance.

Question 2: What is the most effective method for preparing for the GMAT?

Answer: The most effective way to prepare for the GMAT is to continue attempting and solving practice papers. We also recommend that you thoroughly examine the most common GMAT questions.

Question 3: Can I find the GMAT essay topics for practice online?

Answer: You can find the GMAT essay topics for practice online. As a matter of fact, you can visit the GMAC site for mock papers. 

How useful was this post?

Click on a star to rate it!

Average rating 0 / 5. Vote count: 0

No votes so far! Be the first to rate this post.

gmat essay answer examples

People also liked

IELTS success

IELTS success| Tips to master each section

IELTS Grammar

Importance of IELTS grammar | Tips and tricks

TOEFL and IELTS

TOEFL and IELTS | Role of contextual & academic words

simulated tests

Simulated tests | Opportunity to overcome challenges

GRE exam structure

TOEFL, IELTS, & GRE exam structure | Key differences 2024!

GMAT exam

Ace the GMAT exam | Navigate quant & verbal difficulties!

Leave a reply cancel reply.

Your email address will not be published. Required fields are marked *

Please enter an answer in digits: sixteen + 2 =

Start your journey with iSchoolPrep

Need help with your Test Preparations? Contact Us for more details

gmat essay answer examples

Inquire Now

Get e-books, expert guidance, live classes and more....

  • Skip to content

GMAT Prep Courses & Tutoring

Free GMAT Practice Questions

Practice makes perfect—or at least that's how the old saying goes—and it certainly applies to our free GMAT practice questions. Our Free Practice Questions are designed to give you the thorough understanding of how to go about solving a problem that you crave. Our thorough explanations show you what to expect from each GMAT question, detailing question-specific hurdles and common traps. Thankfully, our practice questions provide a wide variety of question types spanning across all sections, securing an abundance of insight-turned-strategy you can implement on test day to turn into high-scoring gold.

Manhattan Review prides itself in providing quality free practice questions to all prospective students, so please take a crack at the 52 free questions we have to offer as a courtesy to all GMAT learners. In the end, the only way to find out where you need your score to be is to discover where it currently is. Take advantage of this free resource that's sure to help you along your way to a high score.

You have not answered any question so far. You can answer all questions in a row (click on "All Questions") or only all questions of a particular section (click on that Section) or a single selected question (click on that Question).

GMAT is a registered trademark of the Graduate Management Admission Council (GMAC), which is unaffiliated with and does not endorse this website.

GMAT Prep Online Guides and Tips

2000+ gmat sample questions: practice every question type.

gmat essay answer examples

Practice questions are an essential part of any GMAT prep. But how do you find the best GMAT sample questions?

In this article, I’ll address what to look for in GMAT practice questions, the pros and cons of both official and unofficial GMAT test questions, and tips on creating an effective GMAT study plan using these resources.

What to Look for in GMAT Sample Questions

There are a wide variety of GMAT sample questions out there. How do you know if you’re using quality practice ones? These are a few of the characteristics of good GMAT practice questions, whether they’re official or unofficial.

#1: Same Format and Question Types as the Real GMAT

Once you start taking GMAT practice tests, you’ll get a feel for how GMAT questions should look: how long they are, what they look like on the screen, the topics they tend to cover, and the style in which they’re written, for example. In every section, there will also be a mix of question types: the quant section always includes both data sufficiency and problem-solving questions, while the verbal section always includes a mix of sentence correction, reading comprehension, and critical reasoning questions.

The GMAT example questions you choose as you prepare for the test should be as similar as possible to real questions written by the Graduate Management Admissions Council (GMAC). The closer you can get to simulating real testing conditions, including everything from visuals to question content, the more comfortable you’ll feel on the day of the test. Also, answering GMAT practice questions that are similar to those you’ll encounter on exam day will help you gauge your progress and potential GMAT score range accurately.

Not sure how or what to study? Confused by how to improve your score in the shortest time possible? We've created the only Online GMAT Prep Program that identifies your strengths and weaknesses, customizes a study plan, coaches you through lessons and quizzes, and adapts your study plan as you improve.

We believe PrepScholar GMAT is the best GMAT prep program available , especially if you find it hard to organize your study schedule and don't want to spend a ton of money on the other companies' one-size-fits-all study plans.

     Improve Your GMAT Score by 60 Points, Guaranteed     

#2: Computerized (and Hopefully Adaptive)

The GMAT is a computerized adaptive test (CAT), which means it adapts to the test taker’s skill level in real time to determine both their score and the mix of questions they’re given. Ideally, GMAT sample questions should follow this format, so you can be best prepared for what you’ll see and experience on test day. Practice resources in CAT format will usually advertise that fact prominently. If they don’t, they’re likely not in the format of the official GMAT.

Obviously not every resource you use will be in CAT format—for example, if you’re using a book to do drills on specific question types. But a sizable portion of your prep should be done using practice questions in CAT format.

#3: Accurate Difficulty Levels

The GMAT always includes a mix of easy, medium and difficult questions. How many you receive of each will depend partly on the computerized adaptive test: You’ll start with ‘medium’ questions, and if you answer those correctly, you’ll be given more difficult questions on average; if you answer them incorrectly, you’ll get easier questions on average. However, every test taker will receive some questions at all three difficulty levels.

Your selection of practice GMAT questions should be at the same difficulty level as the real GMAT test questions. As you prepare, you’ll be able to tell if your sample GMAT questions seem too easy or too difficult on average. Practicing with either too-difficult or too-easy questions might give you an inaccurate picture of your probable ultimate GMAT score, and could also leave you underprepared for the real GMAT.

#4: Organized by Skill and/or Difficulty

You should target your weaknesses when completing GMAT practice questions. You may struggle more with data sufficiency questions than with problem-solving questions on the quant section, for example, or you may do well on most geometry questions but wrestle more with trigonometry and algebra. Alternatively, you may have mastered easy questions across the board and might need to focus on medium and/or difficult questions instead.

Resources that offer customizable sets (based on question type, skills tested, and difficulty level) of GMAT practice questions, or that separate the questions according to those categories, will help you study more effectively and in a more organized fashion than ones that aren’t labeled according to type or that are randomly compiled. The more specifically each question is categorized, the more effectively you will be able to form a study plan that hones in on your specific problem areas.

Computerized adaptive practice questions will help you prepare most effectively for the GMAT.

Official GMAT Practice Questions

The Graduate Management Admissions Council (GMAC) releases a variety of practice resources that use official GMAT sample questions, either written by the test makers themselves or adopted from retired exams. For each resource, I’ll go over what’s included, its strengths and weaknesses, and the best way to make use of it in your prep.

#1: GMATPrep Software

The official GMATPrep Software is the best resource for GMAT practice questions available. It contains two full-length simulated computerized adaptive tests . Everything is written by GMAC, so you know you’re getting the real thing. You can take the two practice tests as many times as you like, but you’ll likely repeat some of the same questions if you take each one more than once.

You can customize your sets of practice questions, choosing 1-15 questions of each type (critical reasoning, sentence correction, and reading comprehension, for verbal) and difficulty (easy, medium, or difficult). In addition to the questions on the two practice tests, there are 90 additional practice questions available on the software.

The software also includes a detailed breakdown of GMAT question types and strategies, as well as a review of skills you’ll need for the quant section.

  • It’s free for registered users of mba.com. Just create an account.
  • The practice questions are identical in length, format, style, content, and visuals to what you’ll see on the GMAT.
  • Answer explanations are step-by-step and in-depth, and you can go back and review or drill questions you previously got wrong as many times as you like.
  • Detailed performance reviews break down what you’re getting wrong so you can hone in on your weak spots, including specific question types and skill sets
  • Not much! The GMATPrep exams are great GMAT example tests. There aren’t many resources that can beat official questions under simulated testing conditions, and it’s free.
  • However, there are only two full-length practice tests, so you’ll likely need to supplement these practice questions with other resources.
  • Also, while the quant review is great, there isn’t a similar resource available on the software for the verbal section.

How to Use It:

  • It’s best to start off your GMAT prep by taking one of the two full-length simulated tests on the GMATPrep software. Since the practice questions and score report are so realistic, the results will serve as a good barometer of your starting point.
  • After you complete the rest of your prep with other practice questions, you should take the second GMATPrep practice test a few weeks before you take the GMAT. This will give you a good idea of where you stand before exam day.

#2: GMATPrep Question Pack 1

An official addition to the GMATPrep Software, the GMATPrep Question Pack contains 404 additional official questions.

You can create custom sets of practice questions of any length based on your needs. Each set of questions can be completed in either ‘exam mode’ (timed, under simulated test conditions) or ‘study mode’ (untimed). The Question Pack will cost you $29.99 to download.

  • This resource contains practice questions that aren’t available in any other official prep materials.
  • Your performance reviews for these practice questions will be integrated into your overall progress reports in the GMATPrep Software , allowing you to get an even more accurate picture of what your needs are in terms of prep.
  • There isn’t much of a downside to official questions as a supplement to your overall prep.
  • Once you’ve taken a few practice tests and figured out your weaknesses, create custom practice question sets that target your difficult spots (particular sections, question types, or question difficulty levels). These will serve as drills to beef up your skills in those areas.

GMAT customizable question banks allow you to work on your timing.

#3: The GMATPrep Exam Collection

These are official additions to the GMATPrep Software. You can only get them once you’ve downloaded the original free software.

Each Exam Pack contains 90 additional practice questions (30 quantitative, 45 verbal, and 15 integrated reasoning), as well as two full-length computerized adaptive GMATs. Each Exam Pack costs $49.99.

Want to improve your GMAT score by 60 points?

We have the industry's leading GMAT prep program. Built by Harvard, MIT, Stanford, and Wharton alumni and GMAT 99th percentile scorers, the program learns your strengths and weaknesses and customizes a curriculum so you get the most effective prep possible.

Try PrepScholar GMAT for 5 Days Risk-Free.

  • In-depth diagnostics will let you know how you’re doing in comparison to your peers on every subsection of the test , as well as how your pacing could improve on each question type.
  • Answer explanations offer step-by-step, detailed analyses of all practice questions.
  • Not much (official GMAC questions are always helpful), except that they’re a bit pricey.
  • These practice questions are a great first step after you take an initial diagnostic practice test, particularly if you’re still unsure of what exactly your weak spots are.
  • You can use the two full-length CAT GMATs to gauge your progress throughout your prep. Alongside the official GMATPrep Software, you’ll have four full-length tests, so you can space them out to regularly check up on how your score is improving.

#4: GMAT Write

GMAT Write is a fairly new tool released by the makers of the GMAT, meant to help you with the analytical writing assessment. It includes two unique sample prompts for the analytical writing assessment section.

GMAT Write will time your essays (30 minutes each, just like on the real exam) and score them in real time according to the GMAC rubric. It costs $29.99 to download.

  • The visuals and experience of writing the essay are highly realistic.
  • The sample questions are written by GMAC, so you know they’re reliable.
  • GMAT Write scores your essays according to the same criteria used on the real GMAT— ‘analyzes the issue,’ ‘supports ideas,’ ‘organizes a coherent idea,’ and ‘language control’ — in real time. It’s the only official GMAC tool that will score your sample essays.
  • There are only two unique exam prompts included, which isn’t necessarily a lot of practice for $29.99.
  • You won’t get any detailed feedback , just scores in each of the four categories used to assess GMAT essays.
  • Use it alongside other practice tests. Many other resources only have integrated reasoning, verbal and quantitative practice questions, so adding a prompt from GMAT Write to your practice test will help you more closely simulated the experience of the actual GMAT.

GMAT Write will help you prep for the Analytical Writing Assessment.

#5: The  GMAT Official Guide 2018

The GMAT Official Guide 2018  is a comprehensive resource for GMAT prep. Included in your purchase ($19.95) is access (both online and in print) to over 900 official practice questions, access to an accompanying site where you can customize sets of practice questions, and online videos with plenty of GMAT tips and strategies.

  • The questions are written by GMAC, so they’re high quality.  The online practice questions mimic the visuals of the real GMAT.
  • The practice questions are organized in order of difficulty.
  • The answer explanations and introduction sections (where key GMAT concepts get broken down) are too complex for non-advanced students. This is especially true for the quant and sentence correction questions.
  • The online question bank only allows you to save ten practice sessions at a time, so if you plan on doing more, you’ll need to delete them before starting a new session.
  • The online question bank is great for regular practice sessions. The question sets are customizable, and you can review them at any time.
  • The book itself is a good resource to start your practice with after using the GMATPrep software as an initial diagnostic tool.

#6: IR Prep Tool

The Integrated Reasoning (IR) Prep Tool was introduced in 2014 to aid students specifically in the integrated reasoning section.

The tool includes 48 integrated reasoning questions total: 10 graphics interpretation questions, 15 multi-source reasoning questions, 15 two-part analysis questions, and eight table analysis questions.

You can create customized practice question sets, focusing on all question types or just one. You can also set the difficulty level for each practice question set. Once downloaded, you have unlimited use of the prep tool for six months. The IR Prep Tool can be downloaded for $19.99.

  • A detailed ‘help’ section breaks down each integrated reasoning question type for you and gives tips on how to approach it.
  • A time management tool tracks your average time on each question type and difficulty level, allowing you to work specifically on your pacing.
  • The IR Prep Tool doesn’t give you a specific score estimation. Your official IR score will be between 1 and 8, but the IR Prep Tool gives you a score between 0 and 100 based on your performance and time management.
  • There aren’t many resources out there specifically aimed at the integrated reasoning section, a frequent problem area for students, so this is a goldmine for students who have trouble with it.
  • Use it to target and break down the integrated reasoning section into manageable chunks, particularly if you get overwhelmed by the fact that each question has multiple components.

#7: GMAT Focus Quantitative Diagnostic Tool

The GMAT Focus Quantitative Diagnostic Tool consists of a 24-question quantitative test (12 data sufficiency questions and 12 problem-solving questions ) that uses real questions from retired exams. It’s computer adaptive and follows the style and format of the actual GMAT quant section.

You can start and stop the test manually, but you have 45 minutes total to complete the practice questions.

Want to Identify YOUR GMAT Strengths and Weaknesses?

Our proprietary GMAT Diagnostic Assessment creates a customized study plan for you that takes you from registration all the way to test day! It is included with every account and proven to significantly maximize your score .

Get your personalized assessment as part of your 5 day risk-free trial now:

Get Your Free GMAT Diagnostic Assessment Here

When you complete the test, the Focus Tool provides detailed answer explanations and an assessment of your strengths and weaknesses in terms of quant concepts.

Four unique tests are available. One test costs $29.99, and a three-test package costs $79.99.

  • The GMAT Paper Tests are the only other GMAT practice resources that use real retired GMAT questions, making the Focus Tool the only computerized adaptive resource to do so. So you get the best of both worlds with this one: real practice questions from previous GMATs and simulated testing conditions.
  • The detailed assessment of your performance, including a possible score range (more accurate than an exact score, since no one practice test will give you a perfect prediction of your ultimate GMAT score) and a breakdown of your performance by question difficulty, will tell you exactly which quant skills you need to work on before the GMAT.
  • It’s a bit pricey for just 24 questions each.
  • It’s not tied into the other GMAT prep resources, so your results won’t be integrated into your GMATPrep performance reports.
  • After you complete one of the GMATPrep exams, this tool will help you hone in on the quant section . You’ll get an overall idea of your strengths and weaknesses in quant from the GMATPrep Software, but this tool will help you get much more specific about which skills to target in your quant prep.

The Quantitative Focus Diagnostic Tool will help you hone in on the intricacies of the GMAT quant section.

#8: GMAT Paper Tests

The GMAT Paper Tests are real retired GMAT tests , written by GMAC. Each set (I, II, and III) contains three official GMAT tests, answer sheets, and a guide for converting your raw score into a scaled score. They’re available in downloadable PDF form. Each set costs $29.99.

  • Most of the questions in these tests aren’t available in any of the other official GMAT practice tests or resources, so they’re especially good if you want some extra practice. This is a big plus: It’s somewhat difficult to find GMAC-authored questions that don’t overlap with other resources.
  • Official questions are always a big positive in terms of GMAT prep!
  • They’re on paper! You’ll have to score them yourself using the answer sheet, and it’s not computerized (obviously) or adaptive, so it doesn’t simulate any of the testing conditions.
  • No answer explanations are included, just the answers themselves.
  • You may want to order these if you’re a test-taker with a disability that may require an accommodation and using paper tests at home is easier for you during prep time.
  • The GMAT Paper Tests might also serve you well if you’ve used all the other official sample GMAT questions and you want some additional prep. However, don’t rely on them to give you much info on your likely final GMAT score, since they aren’t in the official GMAT format. Use them to work through practice questions in your prep sessions instead.

Best Unofficial GMAT Practice Questions

Official GMAT questions are great, but they aren’t the only GMAT prep materials out there. There are also a variety of free and paid resources that offer quality GMAT practice questions. Here, I’ve included practice tests, books, and online question banks. For each unofficial resource, I’ll address how you can access it, its pros and cons, and the best way to incorporate it into your GMAT prep.

#1: Veritas Prep

You’ll need to create an account to access Veritas Prep’s GMAT Question Bank, but once you do, it’s completely free.

The Question Bank offers hundreds of realistic sample GMAT questions: quant, verbal, and integrated reasoning. It includes example questions of every type within those categories. Before you start your practice session, you can choose which kinds of questions you want to see. With one account, you’ll only see each practice question once.

Veritas Prep doesn’t give you an official estimated score, but it does give you an overall percentile , which measures how you did on a particular question set compared to other students who answered the same practice questions.

The Veritas Prep Question Bank is ideal for regular practice throughout your prep, particularly if you have varying amounts of time (you can do a short or lengthy session) and want to drill certain skill sets or question types . The questions from the bank aren’t in CAT format, but you can access one free computerized adaptive test through Veritas and six paid ones. Veritas Prep questions are some of the closest you’ll find to official GMAC-authored GMAT example questions.

Kaplan has a wide variety of GMAT resources, including in-person and online prep courses, but its GMAT Premier 2018  is its flagship GMAT prep tool. Your purchase includes access to over 1,000 GMAT practice questions, six full-length practice tests (one is in the book, and five are computerized adaptive tests available online), video tutorials, an online question bank with 200 GMAT sample questions and customizable quizzes, and a mobile app that allows you to study on the go.

GMAT Premier also includes a special section of forty advanced quant practice questions and in-depth answer explanations, which can help if you’re looking for a challenge in the quant department. The Kaplan practice questions are a bit harder than the ones in Princeton Review, but the verbal questions in particular still leave something to be desired in terms of difficulty level. The sentence correction questions in particular are easier than real GMAT questions. The Kaplan quant questions fare a bit better in terms of GMAT comparability.

Like Princeton Review, Kaplan is a good resource if you’re looking for a fairly solid overview and breakdown of the exam to start your prep (after you take your first diagnostic GMATPrep test). It’s not going to be as helpful if you’re looking for a comprehensive guide to the GMAT or a massive boost in your score.

#3: Manhattan Prep

With an account, you can take one full-length computerized adaptive Manhattan Prep test for free. Six additional ones are available for $49.99, which gives you one year of unlimited access.

You can take the Manhattan Prep practice GMATs timed or untimed, and you can take complete or partial practice exams depending on your time limit during prep. After you take all six, you can reset the questions to take more. Each test includes an in-depth assessment of your performance that analyzes your strengths, weaknesses, timing, and the difficulty levels you’ve mastered.

Manhattan Prep questions are mostly quite realistic in content and difficulty level. Though the visuals aren’t perfect, they are not too far from what you’ll see on the real exam.

I’d also like to point out one of Manhattan Prep’s best resources for students looking to challenge themselves in the quant section: Their GMAT Advanced Quant book includes strategies for tackling the trickiest quant questions and over 150 realistic practice questions. This is a great resource if you’re already doing well in the quant section and looking to bump yourself up to the next level.

#4: PowerScore

PowerScore’s main claim to fame is its trilogy of GMAT Verbal Bibles: Critical Reasoning, Sentence Correction, and Reading Comprehension, along with the PowerScore Verbal Bible , which covers the entire section. The books are available for $24.99 each or $69.99 as a trilogy.

PowerScore does an excellent job of breaking down each question type within the verbal section (a great resource for non-native English speakers and anyone who struggles with verbal questions), and the sample GMAT questions included in each book are consistent with the length, difficulty, and content of those on the GMAT. Though of course the practice questions here aren’t computerized or adaptive, each book does include access to a companion website that includes extra drills and practice questions.

The main downside to PowerScore is that, while the books offer great strategies and explanations of various question types, there aren’t a substantial number of actual practice questions . The Verbal Bible , for example, contains 31 critical reasoning questions, 77 sentence correction questions, and only eight reading comprehension questions. So while these resources are great reads for test-takers who are looking for in-depth answer explanations, they’re not fantastic for substantial extra practice or regular prep sessions.

The PowerScore Verbal Bibles have excellent grammar guides to help you with sentence correction questions in particular.

#5: 800 Score Test

Providing a username and password will get you one full-length test on 800 Score Test for free. You can download five more for $39.95, which includes a comprehensive quant and verbal review, ten sample essay questions, access to online help, and in-depth video explanations of practice questions.

The visuals at 800 Score Test aren’t much like what you’ll see on the GMAT, but the practice questions themselves are high quality, particularly in the quant section. The verbal practice questions veer a little ‘off’ and aren’t as close to the real GMAT verbal section as the quant questions are, so if you’re especially looking to focus on verbal practice questions, this might not be your best bet.

The 800 Score tests are in CAT format, though, so even though the visuals are a bit lacking, they’re not the worst resource if you only want practice questions that simulate real testing conditions. Their Test Pacer tool lets you know what question you should be on to finish in time, which is very helpful for students who have trouble with consistent pacing.

#6: Princeton Review

Princeton Review’s Cracking the GMAT, available for $21.99 on Kindle, offers a breakdown of all GMAT sections (including integrated reasoning and the analytical writing assessment), along with six full-length computerized adaptive practice exams, over 180 practice questions organized by difficulty level, and drills for each test section. Cracking the GMAT Premier is $7 more and comes with more online resources, like extra practice tests, study strategies, and video tutorials.

Princeton Review offers a decent overview of each of the GMAT sections, though it only offers the grammar and math fundamentals, not anything in the way of more advanced concepts. Indeed, where it misses the mark is mainly in the difficulty of the practice questions: Nearly none of them are as difficult as actual GMAT questions. This is especially true in the critical reasoning and reading comprehension sections. So while Cracking the GMAT may help beginners who aren’t familiar with GMAT basics (especially to start off, since it’s not an intimidating text), it’s not enough for test-takers looking to break 700 or even 650.

#7: GMAT Club

Beyond its helpful GMAT forums, GMAT Club offers a variety of useful GMAT practice materials, including practice tests and question banks.

A few practice tests and question banks are free (no account required), but access to all of their customizable quizzes, bonus questions (over 1,572) and full-length tests will run you at least $79.99 for three months of use.

GMAT Club tests aren’t exactly in CAT format, as they don’t adapt to your performance as you go along, but they do follow CAT ‘logic’ and include a mixture of difficulty levels and question types that is consistent with what you’ll usually encounter in a computerized adaptive test. The quality of the visuals and practice question content is high, with practice questions closely resembling GMAC ones.

Difficult quant questions are one of GMAT Club’s specialties, which is important as many of the other top practice resources (Princeton Review and Kaplan, for example) trend a little easier: They claim that hundreds of their quant questions are at the 700+ scoring level. So if you’re looking to wow your prospective schools with your quant score, this might be a great resource.

#8: Manhattan Review

You have several options for free practice questions and other GMAT prep materials with Manhattan Review. With an account, you can access a fairly high-quality quantitative question bank in PDF ebook form, along with a vocabulary list (good for non-native English speakers) and another free ebook, GMAT in a Nutshell , that serves as an overall guide to the test and top strategies.

The Sentence Correction Guide, downloadable for free, boasts a solid grammar review of the concepts that are more often tested in sentence correction questions, but the sentence correction practice questions themselves aren’t all quite as difficult or complex as the real ones on the GMAT.

Manhattan Review also offers a full-length free practice test, which you can access with an account. It’s a computerized adaptive test. The questions here are good for extra practice, but the difficulty levels and visuals aren’t always consistent with the actual GMAT.

Vocabulary practice is an integral part of GMAT prep for non-native English speakers.

3 Top Tips for Creating a GMAT Study Plan

Once you’ve selected some practice GMAT questions, it’s time to set up a study plan. It’s important not just to find the best prep materials for you, but to use them to your advantage. Here are a few tips for using your prep materials as effectively as possible:

#1: Use the Two Official GMAC Tests on the GMATPrep Software First and Last

You should start your prep with one of the two full-length GMATPrep Software tests. Since they’re computerized adaptive tests with accurate visuals and questions written by GMAC, they’re the best possible indicator of your ultimate GMAT score. The first practice test will give you a good idea of where you stand and how to begin your prep. Unless you purchase more official GMAC practice tests, take the second free test from the software a few weeks before your exam date. This will give you a good idea of your score at the end of it all, and will let you know what to brush up on last-minute before the big day.

#2: Use Different Materials to Build Different Skills

Tailor the practice GMAT materials you select to your particular needs. For example, if you’re strong in quant but weaker in verbal (or a non-native English speaker), you might use Manhattan Prep, the GMAT Focus Quantitative Diagnostic Tool, or GMAT Club practice questions for your quant prep, since they all feature more advanced quant questions. You might start out more slowly in verbal, completing grammar reviews and drills from one of the PowerScore Bibles before delving more into practice questions. Mix and match resources according to your personal GMAT goals.

#3: Note Differences Between the Conditions of Your Practice and the Conditions of the Real GMAT

Not all of your GMAT practice questions need to be (or should be) in computerized adaptive form. Sometimes you’ll need to take your time to work through a particular question, complete a drill, or review concepts. But when you do take a full-length GMAT practice test, stick to the real testing conditions as much as you can: Use double-sided scratch paper, don’t use a calculator on the quant section, take the right amount of time for each section, and take eight-minute breaks between the integrated reasoning and quant sections and before the verbal section.

Some GMAT practice materials allow you to manually time yourself, take longer or unlimited breaks, or stop and start the test. Be mindful of these differences so you don’t learn to rely on conditions (such as a longer break) that won’t exist on the day of your exam.

whats-next-1462747_1280

What’s Next?

Looking for free full-length practice tests in addition to the best practice questions? You can find a list in our guide to the best free GMAT practice tests .

Our complete collection of GMAT practice tests provides a more comprehensive overview, in addition to reviews, of all available GMAT practice tests.

Finally our guide to how long to study for the GMAT will help you set up a time-effective study plan.

Was this helpful? Sign up for FREE GMAT and MBA guides!

Share this:.

  • Click to share on Twitter (Opens in new window)
  • Click to share on Facebook (Opens in new window)
  • Click to share on Google+ (Opens in new window)

gmat essay answer examples

Author: Laura Dorwart

Laura Dorwart is a Ph.D. student at UC San Diego. She has taught and tutored hundreds of students in standardized testing, literature, and writing. View all posts by Laura Dorwart

IMAGES

  1. Sample GMAT Essay Prompt (Topic) and Exemplary Response

    gmat essay answer examples

  2. Sample GMAT Essay Prompt (Topic) and Exemplary Response

    gmat essay answer examples

  3. GMAT AWA

    gmat essay answer examples

  4. Gmat essay help! Good Vocab To Use For The Gmat Essay

    gmat essay answer examples

  5. GMAT Writing Section Examples

    gmat essay answer examples

  6. GMAT Essay

    gmat essay answer examples

VIDEO

  1. How to prepare for the GMAT Essays

  2. HOW TO ANSWER IN GMAT EXAM ? #gmatexam #gmatprep #cat #matexam #xat #mbaentrance #mbaexams #mba

  3. GMAT MOOC Session 2 Critical Reasoning Question Type

  4. Essay writing tips you need to see✍️

  5. How to answer this very important essay question✍️

  6. 800score GMAT Essay Guide

COMMENTS

  1. GMAT Sample Essays

    ESSAY #2: Political organizations that advocate the use of violence to achieve its goals can sometimes lead to destruction and devastation. However, that claim that such groups are detrimental to society does not follow the same line of reasoning. These groups might be of great help to certain sections of society.

  2. Example of a High-Scoring AWA Essay

    Sample high-scoring essay. The financial planning office of Fern Valley University suggests that a fundraising campaign be initiated in order to increase the range of subjects taught at the university and to expand library facilities. The goal is mainly to halt the declining number of students and admissions by addressing these two issues.

  3. GMAT Essay: Format, Scoring, and Tips for the AWA

    The GMAT Analytical Writing Assessment requires you to write an essay, using a basic word processor, that critiques a provided argument. It is the first section of the GMAT and is to be completed in 30 minutes. The AWA is scored by two readers in half-intervals on a scale of 0-6.

  4. The Best GMAT Essay Template to Help You Ace the AWA

    An effective GMAT essay template will include the following features: The structure of your essay, paragraph by paragraph. The kind of content that should be in each paragraph. Varied and sophisticated pre-written sentence stems (as in, the beginning part of a sentence) for the main kinds of flaws.

  5. 7 Essay Tips for GMAT Analytical Writing

    The GMAT Analytical Writing Assessment (AWA) provides clear instructions on how you should plan and write your essay. Kaplan students learn these instructions long before Test Day and do not waste precious testing time reading them while the clock ticks. Kaplan GMAT students learn the Kaplan Method for AWA and the Kaplan template for structuring the essay into paragraphs.

  6. GMAT Analytical Writing: All About the GMAT Essay and How ...

    A GMAT analytical writing sample essay, whether well done or flawed in itself, can help you polish your own GMAT writing and bring your essays to the next level. The important part of reviewing example GMAT essays is in analyzing them (and expert analysis is even more helpful, particularly at the beginning). ... * Answers included in the email ...

  7. GMAT Essay

    GMAT Essay practice question 1. The following appeared as part of an article in a trade magazine: "During a recent trial period in which government inspections at selected meat-processing plants were more frequent, the number of bacteria in samples of processed chicken decreased by 50 percent on average from the previous year's level.

  8. The 6 GMAT Essay Tips That Will Help You Ace the AWA

    Another one of the important GMAT writing tips is to take the time to set up your essay in a clear way. You don't need to write the most interesting or lengthy essay in the world to score well on the AWA section, but you do need to give your essay an easy-to-follow structure. Usually, that consists of an introduction, three to four well ...

  9. GMAT Analytical Writing Assessment (AWA) Section

    The Analytical Writing section on the GMAT (often referred to simply as the AWA) is possibly the one with most diametric opinions. Some people actually look forward to the opportunity to break out of the multiple-choice format and to present their thoughts on paper, while others are terrified by the prospect. Whichever side you land on in this ...

  10. GMAT Analytical Writing Assessment tips

    This essay question has been taken from gmatclub and the answer has been judged as a perfect 6 by the GMAT Write tool. The answer was written by the user 'rish2jain' from gmatclub forum. Takeaways. To score a perfect 6 on the GMAT Analytical Writing Assessment section, it is important that you learn the AWA template and practice as much as ...

  11. How to write your GMAT AWA Essay [Effectively]

    2. On the AWA, you need to be a lawyer, that is, you need to find faults with the given argument. You do not need to be a journalist, that is, you do not need to write about all possible perspectives of an issue. Also remember, that this is an analysis, not an opinion piece. Do not bring YOUR perspective and your opinions into the essay.

  12. PDF 12 EXCLUSIVE GMAT EXAM QUESTIONS

    Answer: D. Explanation: This question asks you to identify a claim that is made in the passage about ozone-depleting chemicals. The best answer is D. The passage, written in 1996, states that the rate of increase in amounts of most ozone-depleting chemicals reaching the atmosphere had been reduced since 1987.

  13. GMAT AWA Example Essay

    GMAT AWA Example Essay. Published November 7, 2012 by Magoosh. 2. See an ideal sample argument essay presented for this AWA prompt. In the previous post, I demonstrated some brainstorming and identified six objections to this argument. I then selected three of them as the basis of the essay that follows. This is one way to go about writing the ...

  14. Sample Questions

    Exam Structure Exam Content Sample Questions Test Security GMAT History. Before you register for the GMAT Focus Edition, get a preview of the types of questions you'll encounter on the exam. Remember: You can learn more about section and question type at Exam Content.

  15. GMAT AWA: Issue Essays and Sentence Structure

    Step 1: Understand the Issue. The first step towards strong essay examples is to understand the two sides of the issue. You must either strongly agree or strongly disagree with the presented issue. Unlike real life where most of your opinions are probably a mix of gray, the GMAT Issue essay requires you to take a strong stand on one side of the ...

  16. GMAT Sample Questions

    Below you'll find examples of how you'll be asked to use a chart, graph, or table to answer questions on the Integrated Reasoning section. Sample Integrated Reasoning Questions. Item 1: Andre is buying gifts for his office staff. He wants to spend exactly $280 and he can buy either sweatshirts, which cost $22, or baseball caps, which cost $26.

  17. GMAT AWA Topics

    Sample Prompt 5. The following is part of a business plan created by the management of the Megamart grocery store: "Our total sales have increased this year by 20 percent since we added a pharmacy section to our grocery store. Clearly, the customer's main concern is the convenience afforded by one-stop shopping.

  18. How to Approach Every GMAT Essay Topic: Analysis and Tips

    Claim: The claim is the assertion or conclusion of the argument. In GMAT essay prompts, the main claim is often spelled out for you, preceded by a term like "thus" or "therefore.". There can also be sub-claims that support the broader, overall claim. Evidence: Claims are often supported in turn by evidence —facts, statistics, and ...

  19. PDF Analysis of an Argument Questions for the GMAT® Exam

    This document contains most Analysis of an Argument questions used on the GMAT® exam. Each question is followed by this statement: Discuss how well reasoned you find this argument. In your discussion be sure to analyze the line of reasoning and the use of evidence in the argument. For example, you may need to consider what questionable assumptions

  20. GMAT Essay Template

    All in all, a GMAT essay template is a tool that ensures that you have drafted an effective and coherent essay. GMAT Essay Template: A Sample Template . Most AWA essays that are scored between 5 to 6, have followed a similar template. Keeping this in mind, let us take a look at what an ideal GMAT essay template should contain. ...

  21. How to Write a Powerful MBA Essay—With Examples

    3. Get Vulnerable. Most MBA admissions essay prompts are written with the goal of getting to know as much about you as possible in the shortest number of words. To do that, you're going to have to share real things from your life — to get personal, intimate, and vulnerable. Do not shy away from this.

  22. Sample AWA Prompt and 6/6 Essay

    Sample AWA Prompt and 6/6 Essay. One of my (anonymous) students wrote a 6/6 essay (as graded by GMAT Write) in response to an official GMAC essay prompt, and has kindly allowed me to share his work on GMAT Club for the benefit of all. The following appeared in a memorandum from the director of research and development at Ready-to-Ware, a ...

  23. GMAT Essay Topics

    The GMAT essay is unlike most of the essays you've written in college. The GMAC provides a list of the most formal GMAT essay topics asked in the exam. Ensure to practice the essay topics every day. The more you practice, the better you get at it. Answer mock GMAT essays. It will help you grasp the technique of writing excellent essays.

  24. Free GMAT Practice Questions with detailed Explanations

    Our Free Practice Questions are designed to give you the thorough understanding of how to go about solving a problem that you crave. Our thorough explanations show you what to expect from each GMAT question, detailing question-specific hurdles and common traps. Thankfully, our practice questions provide a wide variety of question types spanning ...

  25. 2000+ GMAT Sample Questions: Practice Every Question Type

    You can customize your sets of practice questions, choosing 1-15 questions of each type (critical reasoning, sentence correction, and reading comprehension, for verbal) and difficulty (easy, medium, or difficult). In addition to the questions on the two practice tests, there are 90 additional practice questions available on the software.

  26. MBA Essay Insight Webinar Series Summer 2024

    Get essay tips straight from admissions representatives at leading business schools during our upcoming MBA Essay Insight Webinar Series! Dates: July 10, July 17, July 24 & July 31. Each session will start with a 60-minute roundtable panel, hosted by Clear Admit Co-Founder Graham Richmond, and admissions reps will share some insight into their ...